Monday, August 10, 2009

Space Warfare V: Laser Weapons

Beams are the classic science fiction space weapon par excellence, ever since HG Wells' Martians zapped Edwardian England at the dawn of SF. For half a century they were almost pure magitech. Then came lasers, and laser weapons have now zapped targets in tests. What's more, they are almost precisely 'heat rays,' just like the ones in the pulps.

Over short distances, relative to the length of the laser itself, laser beams cheerfully ignore the inverse square law that governs ordinary light sources. But thanks to diffraction, over long distance they are effectively subject to it. The formula for the spread of a laser beam (via Atomic Rockets, of course) is a close cousin to the formula for telescope resolution:

RT = 0.61 * D * L / RL

where:

RT = beam radius at target (m)
D = distance from laser emitter to target (m)
L = wavelength of laser beam (m, see table below)
RL = radius of laser lens or reflector (m)

So an ideal diffraction-limited laser zapping in the near IR, with a wavelength of 1000 nanometers firing through a 2-meter telescope, has a spot size of 12.2 cm at a range of 100 km. (Before you break out your calculators, remember that the formula uses radii while my example uses diameters.) If your laser has an average power output of 1 megawatt, each square centimeter is getting hit with about 8.5 kilowatts – about 850,000 times the intensity of sunlight at Earth's surface. The target surface will get very hot, very quickly.

The most refractory material we currently know of, graphite, requires some 50 MW MJ [oops, energy = megajoules, not power = megawatts] to vaporize 1 kg – roughly the energy of 12 kg of TNT – and has a density of about 2.2 g/cm3. Cutting to the chase, our beam will burn through it at not quite a millimeter per second. Most metals are much less resistant to heat, so the laser will burn through metal hulls way faster). But if you substitute a 5 meter mirror – or a 400 nanometer beam, at the short end of the visible spectrum – you'll burn a smaller hole at half a centimeter per second. Or it will have the same spot size and burn rate at 250 km.

Lesson: For a given beam power, the bigger the mirror and shorter the wavelength, the greater the effective range. And lasers cannons, at least in the classical IR-visible-UV band, probably won't look much like guns, but perhaps more like a TV satellite dish.

Let's get a bit more SFnal about it and specify a 100 nanometer UV laser firing through a 10-meter telescope, with beam power of 1 gigawatt and range of 5000 km. Our spot size is unchanged, but each square centimeter is now getting hit with about 8.5 MW, and you'll burn through a meter of graphite in a second. This is some serious zapping. Or you can achieve a 1 mm/second burn rate at 160,000 km, more than half a light second.

Of course there is a tech challenge or two: Operating a laser cannon is loosely comparable to mounting a jet engine at the eyepiece of an observatory grade telescope. You will produce waste heat greater than beam power, probably several times beam power. But all this merely makes it difficult, not impossible. Real lasers presumably won't be as good as ideal ones, but there's no inherent reason why they couldn't come reasonably close to diffraction-limited performance.

Venturing further into SFness, if you make it a 0.1 nm X-ray laser with a 10-meter aperture you now achieve the same spot size and beam intensity at 10 million km. Since that is half a light minute, the target now can dodge, and because X-ray telescopes require an enormous focal length, your laser - and therefore the ship carrying it - may have to be, oh, perhaps 8 km long (see comment thread).

So much for laser basics. Now for the consequences.

If you can see it, you can zap it, and vice versa. As noted above, laser spot size is closely related to telescope resolution. If you can focus the beam to a couple of dozen centimeters, that is also the resolution your sensors can gain, simply by looking through the telescope between pulses. Which means that lasers of this precision don't just score random hits like World War I battleships; they fire at specific points on the target surface. (If mechanical or thermal limits preclude this precision, you won't get the penetrating burn-throughs described above, just scars burned along the hull surface.)

Thus the objective won't just be to blast an enemy ship but to mission kill it by zeroing in on critical systems – such as armament. In a laser battle, if you can hit the other guy effectively at all you can shoot the gun out his hand. But it gets better. What happens when two lasers are zapping each other? Their targeting optics are pointing straight at each other – so the optics concentrate the incoming beam right onto the laser itself. I have no idea what the effect is, but it could easily be dramatic. Laser engagements lend themselves to a mutual eyeball frying contest. Whoever zaps first, probably wins.

But there is another and even more curious implication of laser combat. So far I've been talking about beams concentrated down to blowtorch intensity, kilowatts or metawatts per square centimeter, able to burn right through refractory materials by heating the surface to thousands of degrees K. But what about mere scorch intensity? Say, the 50 watts/cm2 that causes primary thermal burns to humans and sets paper on fire. This won't burn through armor, but it will likely burn out delicate components such as sensor elements, or at any rate saturate and 'dazzle' them.

Thus laser weapons can blind the enemy, temporarily or permanently, at much greater range than they can do serious physical damage to structures. Our first modest laser has a scorch range of 1300 km; the more SFnal one a scorch range of 2 million km … and the jumbo X-ray laser has a scorch range of 2 billion km, about 14 AU. Spot size (and targeting resolution) is wider by the same proportion, dozens of meters. More rugged sensors are the solution, but it seems likely that weapon lasers can dazzle or blind targets at several times the range at which they can burn through armor.

Realistic [TM] laser combat thus has rather little in common with the Hollywood image of gyrating ships zapping each other at smoothbore-cannon range. I'd argue that laser warcraft, like tanks, will typically have a single main weapon in order to provide it with the largest cost-effective optics and thus longer range. This may well be 'keel-mounted,' aimed by orienting the spacecraft (though the optics will likely provide for vernier adjustments).

Yes, this precludes maneuver while firing – but unless you're fighting at Stupendous Range, more than a light second, you can't dodge a laser beam anyway, while at ranges of hundreds or thousands of km tactical maneuver won't make much difference on the time scale of zapping. Railroad guns don't fire while the train is moving, and laser cannons will plausibly have equivalent constraints.

So if you want the furball combat effect, or anything close to it, you will need a workaround - such as an engagement in the clutter of orbital space, where the challenge is distinguishing hostiles from civil craft and stations you do not want to zap. Or, long range sensor blinding might produce a strange battle of lasers concealed behind armored ports, taking potshots like spaghetti-Western riflemen shooting from the windows of a ranch house.

The floor is open for discussion.


Related links: Previously in this series, The Gravity Well, Stealth Reconsidered, Space 'Warships', and Mobility. And earlier, my take on Space Fighters.

122 comments:

Citizen Joe said...

A little more fodder.

You can have lens focusing and mirror focusing. Lens focusing really just works for visible light (maybe some UV).

In either case, your lens/mirror needs to be significantly larger than the target point or you'll simply melt your mirror/lens before you even start. In order to magnify the damage on target, you have to start with a larger area at the source.

At extreme ranges, it doesn't take much heat to distort the targeting array. During the final polishing the mirrors for reflective telescopes, they were only able to rub the mirror with their thumb for a few seconds before the heat build up could cause a distortion.

Xray lasers can't be focused with regular lenses/mirror. They need very low angle xray reflectors.

Gamma ray lasers probably aren't feasible since their wavelength makes them susceptible to atomic level variations in the reflectors.

UV lasers are vaccuum spectra, meaning they only really work well in space. Atmospheres tend to block the UV rays. So you'll find that planetary bombardment lasers will be Visible light, possibly seen as they pass into clouds, while space combat will likely use UV for the range without the exceedingly difficult Xray laser issues.

Finally, in no case do you actually SEE the laser. First of all, they are all going in the same direction, so you won't see anything until they reflect off of something and scatter. Second, it probably won't be in a range visible to people.

Here's a fun video talking about lasers:
http://www.youtube.com/watch?v=0AV5aY7HYAk&NR=1

Anonymous said...

Starships as tanks. Just like the real-world equivalent, big, overwhelming at long ranges but fragile at short ranges, and expensive. With realistic automation they might even have similar crew numbers. And again, just like the real-world equivalent, they'd be almost useless in built-up areas.

Ian_M

H said...

Interesting:
Just some coments:

Why do laser "cannons" have to be fixed?, a turret like the one mounted oin the YAL-1 ( a technology demonstrator, it consists of a Boeing 747 equiped with a laser turret in order to destroy balistic missiles). Such a system would give the ship the ability to react faster to unexpected threats like missiles.

If spacecraft can "blind" each other at a longer distance than they can destroy each other, this opens an interesting possiblity. The first ship who adquires its target can effectively negate its adversary the ability to shoot back, it could use a second weapon in order to destroy its target harmlesly or simply fly past it toward its intended target (a command and controlship or a planet).
A turret based weapon would allow to maintain your target blind even if you move in relation to it.

This last idea however may not be so easy at all if the blinded ship carries missiles with independent target adquisition. More lasers are necessary in order to blind or destroy the missiles.

Citizen Joe said...

The YAL-1 has a hopeful range of 600km. It is necessarily a visible light laser to get through the atmosphere. Assuming adaptation to UV, you're looking at about 10 times that range in space or 6000 km. In space combat, that is pretty much right on top of each other.

The game Traveller uses range bands for combat and those range bands are 30Kkm (1/10th of a light second).

The turret system you suggest might be good for point defense against incoming missiles (which is what the YAL-1 is designed for). However, as a starship weapon, turrets simply won't have the range. And by the time you make them large enough for that range, they don't turn fast enough to track close targets.

The target blinding technique IS very effective, and that may be another excuse for fighters. Acting as forward observers, fighters can report the position of enemy craft while the main guns are essentially blinded.

Unknown said...

Neat overview of continuous burn lasers. What would be the differences (in optics, damage modes, power requirements, peak powers, etc) for pulsed lasers?

Thanks,
-- Pat

Luke said...

Note that for half light minute ranges where the target can dodge, this itself can be used as a weapon. By forcing the target to use vital propellant while dodging, you may be able to mission kill it without ever hitting it.

Also, my original estimates of many kilometer long x-ray laser warships from SF_CONSIM use already out-dated accelerator technology. With modern materials, we can get the laser to around 1/10 the length, so you "merely" have nearly kilometer long death ray spacecraft.

Anonymous said...

Lasers seem almost the ideal weapon for space-based and orbital combat, almost too perfectly if you ask me. Of course the diffraction and heat generation problem does limit the laser's potential so it's not a wonder weapon without flaws. However the idea mentioned in the blog is also interested. The ability to not only burn through a target's armor, but also to blind the sensors either passive or active (I think) of an enemy space craft at greater ranges than at distances to burn or scorch.

This fact alone may legitimize the deployment of multiple, autonomous drones that could be deployed from the mother craft in addition to escort space craft to ensure that the command crew's view of the battlespace isn't completely blinded. Used like forward observes as Citizen Joe notes, though the limitation of other Laser-like systems such as Xrasers, Grasers, and Uvasers that Citizen Joe has also noted to have their own technological problems.

The Turret rotation and orientation Citizen Joe noted does help give a spinal mount laser "cannon", for lack of a better word, and other such main batteries acquire targets without having to maneuver the entire spacecraft to do so. However, that notation about turreted laser cannons having significant lesser range is something to think about. Meaning that for the kind of combat ranges lasers are reputed to have even with diffraction, combat spacecraft will still have to maneuver and reorient themselves in order to line up with the target craft. Me thinks that this might make an excuse to have a kind of Fighter Pilot mentality for the helmsmen of such spacecraft in addition to a kind of Top Gun Orbital Combat School, but that's just me.

And if this article is any indication: http://www.thespacereview.com/article/1438/1
Then there might be good reason to arm or guard these Battlestars with laser weaponry.

And I'll assume that Citizen Joe found that little video on the physics of laser and kinetic weaponry in my post on the "Science Fiction, Hard and Otherwise" blog entry.

Oh which reminds me, while we're on the subject of combat lasers. I have heard of various laser types such as Solid-State, Dye Lasers, Gas and Chemical, Free Electron for starters. I have to ask if there are really any differences and benefits, particularly in how they operate for combat, outside of how these laser types generate their beams of perfect light?

- Sabersonic

Jean-Remy said...

"Their targeting optics are pointing straight at each other – so the optics concentrate the incoming beam right onto the laser itself. I have no idea what the effect is, but it could easily be dramatic. Laser engagements lend themselves to a mutual eyeball frying contest."

Enter good old fashioned gun ports. Well, nothing old fashioned about them. Take your high-refractory graphite, and build a "camera shutter" Use passive sensors or radar to get at least an approximate track on the enemy, then take quick millisecond snapshots with the laser optics themselves to refine your targeting. Then, fire in short pulses, closing the shutter between firing sequences.

It's not a perfect solution, but at least it stops either side from taking the time to target each other with any great precision. In fact I can see laser battles actually going on for longer than the first salvo. To protect your optics you might not even use your high-definition telescope, which happens to be your laser, to target at all: this is the most precious and most fragile element of the entire ship. Without it you just have a multi-thousand ton multi-billion dollar paperweight.

And it doesn't even do that properly since, you know, no gravity.

I can see then laser gunfights primarily as "shooting from the hip" contests. The ECM war is alive and well. Maybe active radars aren't even used either as an active pulse might be used by the adversary to get your range. I can see half blind ships using quick glances at each other and computers to refine probable trajectories and then shooting until they hit something. The ship who takes too long a look could be the one who loses.

And here comes the human element in warfare again. The guy who runs the simulation and trajectory refinements needs the human instinct for his enemy: which way is he going next? Is he going for a bold least-time intercept to get as close as possible with all his optics protected to open a full salvo point blank, for the best chance to score a blind hit (point-blank being 30,000 km) or has he shown to be timid and will maintain a 300,000 km range. The captain of the ship has, of course, has his own challenges: does he take a good long look at his enemy and refine his targeting? Or does he hope to get a lucky shot?

It could actually turn into an interesting war of nerves, especially in a crowded orbit.

Rick said...

First, even if it is a shameless pander, it is also true: This blog gets great comments.

Second, thanks to Anthony Jackson at SFConsim-l for catching a bonehead error. The heat of sublimation for a kg of graphite is of course in joules, not watts: 50 MJ. With luck that's the only out and out fail in this post!


qwert - On fixed laser mounts v turrets, a clarification: It isn't necessaryfor the mounts to be fixed. But other things equal a fixed mount can have a bigger main mirror (or lens), hence longer range, so I think fixed is preferable for the big primary weapon. Laser craft may well carry smaller bug zappers that would be turreted. (Or at least in a rotary mount, whether or not precisely a 'turret'.)

Pat - At the level of generalization in my discussion there's not much difference between CW and pulsed lasers. (Think of the expression 'hosing' for automatic fire.) Pulsed may be preferable, though, to give time for the detritus from each pulse to dissipate before the next, and for a jackhammer effect on the surface you're zapping. Each pulse might be a nanosecond or less, with a pulse rate from a few hertz to megahertz.

Citizen Joe - The problem for fighters as forward observers is that at space laser ranges, the battlestar can swing to blind the fighters, then go on engaging its primary target.

Luke - Nice point that at uber-range, even misses force the target to dance, expending delta v!

Sabersonic - Discussing specific types of lasers is above my pay grade! :-)

Ian - The problem of tanks in urban fighting is indeed parallel to the problems that battlestars designed for uber-range zapping might have in cluttered orbital space.

Jean - And 'shooting from the ranch house windows' is one of those things that could throw the baseline assumptions into a cocked hat.

Sabersonic again - Lasers are indeed 'almost too perfect.' Anticipating a future post, the way to defeat lasers with kinetics is to throw LOTS of stuff. But if you have interplanetary trade, you are pretty much in the business of throwing lots of stuff, and throwing it pretty damn fast.

Citizen Joe said...

Wow, someone else came up with the multikilometer long Xray laser ship? The one I designed was based on the assumption that the D-He3 fusion reactor for the drive would need too much magnetic containment for a circular accelerator, thus they used a linear (probably hyperbolic helical) accelerator. Then slaved a Free Electron Laser to the accelerator and tuned it into the X-Ray range. Actually it was tunable along the whole spectrum, and used for communications as well. Anyway, my point is that the length of the ship was only to accommodate the power source for the drive and used the excess power for the laser. What really gets you the range is the focal array, which I think was like 50 meters.

I think that array could also change mode to a particle beam, which was very close range but much more destructive.

Anonymous said...

Wow, lots of comments...
First: the YAL-1 has two beam paths in its turret, one for the sensor beam and one for the weapon beam; it doesn't shoot when it looks, nor does it look when it shoots. Zapping a laser with another laser would (I should think), either "re-pump" the laser generator, burn the front end optics, or destroy a component that is specificly designed to do just that.
Second: A real laser equiped spacecraft would probably have one laser generator and several turrets (including a high-powered, long-range one on or near the nose), each one optimized for different uses.
In the near-term, I can see several designs being developed and the most useful evolving out of experiance...at one time or another every sugestion in the above posts will probably be used or tested.

Ferrell

Citizen Joe said...

The counter to lasers forcing the expenditure of delta V's is that if you chuck a bunch of stuff at a laser ship, it has to shoot it all, which produces like 4 times the heat that the laser puts on target. You can force a mission kill through overheating the weapon systems/reactor.

I was playing in a Traveller game once where, after killing an assassin, we found his scout ship in a hanger bay. Only it was electrified to prevent access. I ordered the radiators foamed and then a fused tap attached to the hull to force a reactor shutdown. The referee was not pleased.

Anonymous said...

Sabersonic- solid state lasers are best pulsed, gas lasers are best CW, and most lasers have a specific frequency they operate at, but Free Election lasers are tunable; you can operate them at (theoreticlly) any frequency.
I hope that helps!
Ferrell

Luke said...

Citizen Joe:

Here is the post where I worked out many of the details on an x-ray free electron laser
http://games.groups.yahoo.com/group/sfconsim-l/message/55548
but instead of the paltry 15 MeV/m accelerating gradients I assumed, the folks at SLAC are already talking about 130 MeV/m accelerating cavities.

(I was also a bit off on the efficiency - that's the efficiency of the energy recovery process, not wall-plug efficiency, which might realistically be around 30% if you need cryogenically cooled accelerating cavities.)

Changing mode to a particle beam is not really an option for the equipment used to generate the x-ray beam. The electron beam will be nearly useless in space - shape charge effects will cause the beam to quickly spread, and the magnetic fields of the solar system will cause the beam to veer unpredictably. You can't reconfigure the beam to shoot protons (ideally, you would shoot an equal number of protons and electrons in a bunch, giving you a neutral plasma) because protons are much heavier than electrons and would get out of phase with the accelerating fields very rapidly.

Very long range lasers like this have a significant advantage over kinetics. If you try to throw a bunch of stuff at it, it has weeks to shoot it all down at its leisure, since it can start effectively beaming when the kinetics are light minutes away.

Sabersonic:

Chemical lasers can achieve high output powers, on the order of megawatts, and are the most mature of the laser weapon technologies. They typically lase at around 1 micron wavelength, in the near infrared (the more advanced varieties, anyway, older chemical lasers emitted around 3 micron wavelength mid infrared radiation). Unfortunately, they have a severe drawback - they require huge quantities of corrosive, toxic, and unstable chemicals, and produce toxic and corrosive exhaust. They will quickly exhaust their chemical supply, after which they don't lase any more.

Dye lasers are horribly inefficient. Only about 0.1% of the input energy gets turned into a laser beam the last time I checked. They are good for research, where you can tune them to any color you want, but bad for weapons because of their inefficiency.

Solid state lasers are the new most promising thing for laser weapons. They can achieve close to 20% to 30% efficiency, emit at around 1 micron near infrared, can put out hundreds of kilowatts, and operate continuously as long as you supply electricity without producing nasty toxic byproducts (plus, electricity is cheaper than lasing chemicals. Diesel to generate the electricity is also cheaper). Work is underway to increase their power even more, but they are on the verge of being weaponized.

Gas lasers seem pretty much a dead end. Carbon dioxide gas lasers are fairly efficient and reasonably high powered (20% to 30%, and up to several tens of kilowatts), but their 10.6 micron wavelength emission gives them too much beam spread and makes them vulnerable to cascade breakdown if they focus too tightly.

Free electron lasers are also promising. They will be large, but they can potentially have high efficiencies and can be tuned to operate at any wavelength. They are under serious study for naval point defense weapons where their ability to rapidly shift beam color will allow them to choose an optimal wavelength for penetrating sea air and burning down the target. Free electron lasers have been demonstrated with outputs of several kilowatts, it is hoped they can put out several megawatts in the next decade or so.

Citizen Joe said...

Ya, I was figuring electron beams. And yes, the were VERY short range. The net effect is a lightning gun.

While my proposed ship did have essentially turrets as well as the main gun. The turrets had blind spots directly in front and behind the ship (so that you didn't accidentally blow off your tail in a furball). While nobody wanted to be anywhere near the thruster array, getting in front of the ship before the focus was damaging would pose a threat. That leads to what amounts to a particle shotgun, where the lightning tends to ground out into other ships, possibly killing the crews with bremstraulung (sp?) radiation.

Anonymous said...

Ferrel and Luke: Thanks for the info on what would work best as weaponized laser weaponry. Though I have no idea as to what the fug "CW" is suppose to stand for. I'm assuming Combat Weapon until some one correct me.

I had been wondering which laser type would be utilized as what on a near-future battlefield. From what I've been researching (and advised) so far, combat laser systems based upon the Free Electron type could have the dialing ability feature made famous by the Star Trek though with a variety of settings such as "Dazzle" to temporarily blind an enemy combatants long enough to pacify them in addition to a PEP style Stun feature in addition to higher and more damaging levels of laser projection.

Getting back on the track of weaponized laser projectors onboard combat spacecraft, from what Luke has enlighten me on the laser types it would probably be wise for combat spacecraft designers to mount a variety of laser type batteries instead of just one. With Chemical Lasers utilized primarily to mission kill other hostile spacecraft whenever they get the chance, Solid-States to knock out (if not destroy) smaller targets akin to point defenses like the Phalanx CIWS, and Free Electrons for such mission profiles when neither the Chem Laser nor the Solid-State would be appropriate.

Unfortunately, I got two more questions to throw at the posters here. Sorry, my mind works like that.

1) Are Solid States the only laser type that could be used for LIDAR and LADAR systems or are there other laser types?

2) How the *bleep* does one turn a laser into a particle beam?

- Sabersonic

Luke said...

Citizen Joe:

An electron beam would not be much like a lightning gun. In addition to not getting conductive plasma channels in space to carry electric current, you don't get electric discharges unless one spacecraft is at a much different electric potential than another. You certainly don't get glowing bolts or forking paths, just loopy dispersing invisible beams.

This does lead to an additional issue - unless you emit an equal amount of positive charge, using an electron beam weapon will charge your spacecraft until it has the same potential (in volts) as the kinetic energy of the emitted electrons (in eV). At this point, the electron beam will turn around and impact the ship firing the particle beam. The solution, of course, is to either vent some positive ions or absorb electrons from the solar wind to neutralize your spacecraft.

As an aside, electron beams could potentially be effective weapons in an atmosphere, where their charge can be screened by positive charge attracted from the air, leading to magnetic self-pinching keeping the beam together.

Citizen Joe said...

The particle beams are used to generate the photons for the free electron laser. So rather than using the laser, you instead just direct the particle beams out into space.

And I believe that the D-He3 fusion thruster was dumping a great deal of protons out the tail end of the ship.

Luke said...

Sabersonic:

On a near future battlefield, expect free electron lasers to be quite large. You could fit one in a 747 or a cruiser. It may even be possible to miniaturize one enough to fit into a standard shipping container. These are not really the size of weapons you would expect to be useful for dazzling.

For spacecraft, I expect one craft would carry only one weapon laser. It could rapidly shunt the beam to various secondary beam-pointing telescopes as needed. The laser itself would be buried in the bowels of the craft, protected by armor, only the scopes would protrude from the armor. For example, if you had a high powered main laser for zapping other distant spacecraft, you could easily tap the beam for point defense by putting the main beam through a smaller, more mobile scope.

I doubt anyone would use chemical lasers if alternatives were available. My guess is either solid state or free electron lasers will be used (or possibly something like phase locked diode lasers - these would be similar to solid state lasers but high powered diode lasers have been demonstrated with greater than 60% efficiency). A free electron laser lasing at short wavelengths for long range in space could be re-tuned to visible or near visible wavelengths for planetary bombardment.

Any type of laser would work as LIDAR and LADAR. What matters is that you can raster the beam, not how the beam is generated. The tighter you can focus the beam, the higher your resolution, so once again you can resolve the same size features as you can blast.

We were talking about turning a particle beam into a laser. A free electron laser shoots an electron beam through an array of alternating magnets to create light.
http://en.wikipedia.org/wiki/Free_electron_laser
However, you can turn a laser into a particle beam using a mechanism known as wakefield acceleration. This involves using a laser to turn a gas into a plasma and producing strong accelerating field gradients at along the plasma
http://en.wikipedia.org/wiki/Wakefield_Accelerator
Don't expect high efficiencies from wakefield acceleration though - for weapons, you might as well just blast with the laser.

Rick said...

Technical specifics noted for general coolness - I don't have much to add! But I shoulda said that CW means continuous wave, as opposed to pulse.

I like the idea of having one big laser inside the ship, with alternate optics it can fire through - probably a 'maximum caliber' main mirror and secondary defensive optics. It moves the laser weapon farther from the mere analogy to big naval guns.

Rick said...

I forgot to add it's no surprise that more than one person has explored plausible reasons to have spaceships several km long. That's a classic case of an idea with such a high inherent coolness quotient that all you need is an excuse. :-)

Jean-Remy said...

I think I still would have more than one lasing tube per ship, for several reasons.

1/ Cycling: the lasers can cycle/cool down between shots. It would also reduce the wear and tear on each individual tube for the same number of total shots.

2/ Multiple targets: it would allow each ship to engage a larger number of targets at once, or targets coming from completely different vectors trying to outflank the ship.

3/ Concentration of fire: bracket a target with two or more beams, either concentrated on the same area of the ship to put more energy on one point, or disable multiple systems on the same ship.

4/ Multiple roles: One laser could be used for point defense, while another is fired through the main optics at the enemy vessel.

5/ Redundancy: if one of the lasers is knocked out by battle damage, the ship is not out of commission.

Anonymous said...

Luke: Yeah, I just noticed that little size notation on the Free Electron Laser. *knocks on own skull* dumb dumb.

Though Jean Remy does have some points against having a central laser onboard a spacecraft, chief of these being the cooling and redundancy points. Not really sure how the whole laser piping would work without something burning along the way to an exit port that's currently closed. There is a benefit to having a central laser generator powering a weapon battery for a particular weapon class, for lack of better wording. One would have the benefit of having a centralized and well protected source of offensive and/or defensive weapon power of more vulnerable gun and cannon apertures.

Another would allow the much needed cool down time for the laser weaponry, especially if they're not being used while facing off an enemy spacecraft in cluttered Earth Orbit.

Though, after looking back to the Sidearm section of Atomic Rocket, would it be possible to utilize the Laser Bullet into a Shell format. This is brought back from a memory of the fleet combat scene in the third movie of the Star Wars Prequel trilogy which showed the cannons being loaded with and ejecting shells akin to the kinetic ballistic variety.

There are probably reasons against this, such as added mass, complexity in the feed system and the like. However, this kind of laser feed operation would be useful in an open cycle laser cannon with the heat of the laser generation being ejected into space and help ease the onboard heat sink.

As for the multi-kilometer sized orbital spacecraft note by Rick, I just have to wonder if that's such a good idea to put that much investment, monetary and resource wise, into a spacecraft that large dedicated to combat? Granted, it gives a good excuse to stuff them with heavy kill weaponry and legitimize a constellation of escort spacecraft in addition to the intimidation factor towards smaller combat capable spacecraft. However, like the Supercarriers in the US Navy, a loss might as well be equal to loosing an entire Task Constellation.

Luke said...

Jean Remy:

Modern weaponized chemical lasers and solid state lasers and proposed free electron lasers can fire continuously at full power without overheating.

A single laser could still engage many targets if you have multiple beam pointer telescopes. Each scope tracks a separate target and the main beam is divided between them, either each gets a fraction of the full power, or the beam is cycled between scopes at full power. The cycling option allows higher power per target than you could get with separate lasers.

If you use a single laser rather than two smaller lasers that add to the same power, you get the same effect on the target if they are aimed at the same spot. You can aim one large laser at multiple spots with multiple scopes.

One laser can be used for multiple roles, again dividing the beam between pointer scopes. Since the main laser can have a power at least that of the combined power of multiple smaller lasers, it loses nothing by doing so but can concentrate more power on a given target when dedicated to that purpose.

Redundancy is a good argument. However, solid state lasers can be made modular - one of the most advanced weaponized SSLs currently developed is made by chaining together 15 kW modules so their beam powers add. This is inherently redundant - if one gets knocked out, you only lose 15 kW and can keep lasing with the others. I expect you could do something similar with phase locked diode laser arrays. Free electron lasers are more difficult - you get a significant economy of scale, with larger FELs capable of effects that you cannot achieve with smaller FELs (namely, shorter wavelengths). Thus, it makes sense to have one accelerator that boosts electrons to as fast as they can go without splitting your available mass budget into smaller accelerators with less oomph. You can divert the electron beam through separate wigglers - one for anti-spacecraft x-ray beams, one for planetary bombardment visible light beams, but redundant FELs means longer wavelengths and shorter ranges than one big FEL.

Jean-Remy said...

Luke a couple of suggestions to respond to your objections:

First the two/three/four lasing cavities would be tightly bundled near the core of the ship, so they wouldn't be that much more vulnerable to damage than one central tube. On reflection, not that tightly since it would invalidate the back-up in case of damage. Say then as close to the core as possible but with a buffer space between them, like the lithium heat sinks.

Secondly, I would imagine a complex (though potentially fragile which may make it a valid point indeed) I want to say railroad-type shunt system. All the tubes are interconnected, and by simply moving a (set of) mirror(s) to the correct position you could bounce the beam from the different tubes through the same set of optics. Like at a major rail hub, you simply divert the beam down a preselected track. Failsafes would prevent a tube from firing though a close track.

I tend to agree with Anon's last point, however. Nothing says target like a 5 kilometer behemoth with the maneuverability of a beached whale. It's especially problematic if you have to turn the ship to aim the weapon: a two-pronged attack and the gun can only be used against one of those incoming forces.

I like the idea of a 5 km ship with a laser that can reach out and touch someone several billion km away. Convince me that I can't take it down with a dedicated flotilla of frigates and drones massing and costing less in total that the one ship, and I'll buy two, keep the change.

Jean-Remy said...

Luke: Oops previous post was meant to answer anon and not you. My gaffe.

Impressive data on the solid-state lasers! They seem to answer all my qualms in a very elegant manner. I guess I'll have to strip out my FEL's and replace them with a core solid-state one then. I will definitely do my research on those. Thank you.

Anonymous said...

Yeah, that post that has the note against multi-kilometer long combat spacecraft was mine. Forgot to add my name, again.

Though weapon systems with their own heat sinks independent of the central heat sink for the rest of the spacecraft isn't such a bad idea. It greatly extends the operational time for any combat oriented spacecraft before the onboard crew must surrender and allow their craft to cool down. Not really sure how much though if all the waste heat of the entire spacecraft is focused upon a singular heat sink as opposed to individualized heat sinks.

However I really don't think that modern weaponized lasers can be used as an example of how lasers can't overheat as Luke has noted. Unless I missed something, these lasers operated and were tested planetside and there are many ways to drive the waste heat away from the weapon and to efficiently cool the laser to operate at peak efficiency. When one has more than one way to manage waste heat in a combat environment with a thick enough atmosphere, overheating isn't that much of a worry.

Space, however, only has one form of waste heat management last I checked: Radiation. I really don't remember any tests of weaponized lasers in orbit that would demonstrate their performance.

- Sabersonic

Jean-Remy said...

About the heat

I figured even warships in normal operations (orbital patrol) would have large radiators spread out like a windmill. In combat however those would make huge, very visible targets, and very very fragile targets. Destroying those radiators would also force the ship to disengage, and might even fore it to power down and surrender.

When about to enter a fight, the ships would fold those radiators and switch to internal sodium or lithium sump tanks. Those metals have both high specific heat and a low expansion factor. If the battle is short enough the radiators are extended and the heat is sent back from the lithium tanks. If the battle starts to take too long, some of the lithium sump tanks might even be vented to space.

Ubreputz said...

There is another cooling method. It is an open cycle method where you dump your waste heat into something and eject it. If that open cycle heat sump can be used for reaction mass, so much the better. What that means is that smaller crafts (say fighters) can use consumable lasers which pack much more punch into a smaller size, and then dump the heat with the core rather than requiring the massive radiators (which are a liability in combat).

Another trick is to push an ice boulder around with you, using that as a heat sink. In addition to dumping your heat during the energy intensive closing action, the boulder can act as a shield to absorb incoming lasers.

Jean-Remy said...

Ok I've been trying to figure this one out since I read it, but I'm either missing something or there's a mixed metaphor in there but...

Rick said: And 'shooting from the ranch house windows' is one of those things that could throw the baseline assumptions into a cocked hat.

What?

I'm not entirely sure what you mean by that.

Glenn said...

"Citizen Joe - The problem for fighters as forward observers is that at space laser ranges, the battlestar can swing to blind the fighters, then go on engaging its primary target."

Hmm, but isn't that a good reason to have them then? If the fighters/drones posit a credible threat either from simply being forward observers (feeding targeting data) or carry either kinetic or laser weaponry themselves of sufficient threat, then attacking from off angle vectors that require the defending ship to swing off axis from the main enemy mothership essentially gives the enemy mothership the "first zap" capability and helps to waste enemy delta-v (through turns etc...). This in turn may require an attacking ship to have turrets (for multi-vector defense) or carry swarms of fighter/drones themselves dedicated to killing other such attackers. This could cause a further opening up of range between main ships and posit 3 roles for fighters/drones 1) redundant targeting 2) close in defense if armed against missiles/drones 3) outside laser range interceptor role against drones/missiles. The key difference is that the drones don't need to necessarily attack forward, they are more likely in the targeting support role placed far to the sides or somewhere behind the main ship in an effort to force the attacker to swing to engage them.

Question about plasmas (cold plasma, etc...). I've heard varying accounts that plasma sheaths could be used to absorb/deflect certain laser wavelengths. Is this true, and if so what limitations does this impose on laser weaponry?

Luke said...

Re: Heat. The comment about modern weaponized lasers firing continuously was illustrating that a single large laser is at no disadvantage to multiple smaller lasers. A single 10 MW laser would generate as much heat as a 5 MW main laser and 5 1 MW point defense lasers, all could fire continuously as long as the spacecraft had the radiators to handle it. Now, as Anon/Sabersonic mentioned, a separate radiator system for the lasers could be a good idea - you might need to keep the lasers much cooler than the power plant exhaust, and thus coupling the laser coolant to the power plant exhaust fluid would either cook your laser or give you much larger radiators than is necesary for optimum power plant operation.

Personally, I don't like open cycle cooling for lasers - one of their beauties is they have no expendables. However, I concede that for short term engagements dumping heat in the form of hot waste materials might make sense. Note that chemical lasers do this automatically. Trying to dump the laser heat into the exhaust is likely an exercise in futility (or at least making so many engineering concessions as to significantly impact the spacecraft performance), largely because you need to pump the heat from a cooler reservoir (the laser) into a higher temperature reservoir (the main drive exhaust), against the natural flow of heat (and requiring a significant amount of work to do so, which imposes a greater draw on the power plant, which creates more heat ...).

Re: lasers and fighters. Lasers tend to be large pieces of equipment, and thus difficult to put on a fighter. Especially since larger focusing elements let you focus the beam tighter, so a a large craft can zap a smaller fighter with the same power laser from significantly farther away, leading to a sort of turkey shoot while the fighters drift into range. However, there is a way to use fighters to leverage the main laser on your primary beam warcraft: make the "fighters" little more than a drive and a big mirror (a design I call a "mirsle", for mirror missile). Now the primary beamcraft can hang way back. Instead of needing to focus its laser to a 5 cm spot on the enemy to cause damage, it can focus on a 10 meter spot - the size of the fighter's mirror - at 200 times the range, and the fighter reflects and focuses the beam to a 5 cm spot. You get to send cheap fighters into harms way rather than big expensive beam craft with big expensive power plants and big expensive lasers.

Re: plasmas. A plasma can absorb light with a frequency lower than the so called plasma frequency of the plasma. The plasma frequency depends only on the density of free electrons. I go into more detail here
http://www.panoptesv.com/SciFi/Ionization.html
but in brief, singly ionized air will not have enough free electrons to stop most weapon frequencies (such as 1 micron near IR, or anything in the visible, UV, or x-ray). Since getting any fluid with the density of air and the temperature necessary to ionize it around, this does not bode well for using plasmas to defend a spacecraft (in addition, if it did block laser beams, it would blind the spacecraft since light could not get through. It would probably also cook the spacecraft from the radiated heat).

Anonymous said...

Sabersonic-"Ferrel and Luke: Thanks for the info on what would work best as weaponized laser weaponry. Though I have no idea as to what the fug "CW" is suppose to stand for. I'm assuming Combat Weapon until some one correct me."

Sorry! 'CW' means Continuous Wave, in other words, it isn't a pulse but on continuously as long as power is applied. I hope that clears up any misunderstanding I might have caused for you.
Ferrell

Anonymous said...

To expand upon the idea Luke has, a Mirror Drone can also give a kind of loophole in the Orbital Bombardment department of space warfare. It has been mentioned in previous blog entries of the Space Warfare series that orbital combat for spacecraft isn't as superior as it is portrayed in other science fiction mediums to which there is effectively no way for any attacking spacecraft from hiding from planetary ASAT weaponry. All they could do is shoot down or avoid these munitions and the closer these spacecrafts are to the target planet, the higher the chances and shorter the time frame of the craft to be attacked.

However, Mirrored Drones could be used to direct the mother spacecraft's laser cannon that is parked in a higher orbit. The drones may be shot down by retaliatory strikes, but the mother spacecraft will still be able to defend itself from planet bound anti-orbital weaponry and have the time to ready itself from such strikes.

As for the Plasma Sheath idea of shielding for a spacecraft, well there is research being conducted in the generation of Cold Plasma Sheath's that could be used upon vehicles and spacecraft as a way to mask them from microwaves if not outright protect them from laser bombardment of a certain frequency. There is even the suggestion of layering hot plasma in between layers of cold plasma. Not really sure how that'll help.

Though the inability to see through one's own Plasma Sheath for target acquisition would present a problem. I'm not sure how good radio waves can traverse a plasma bubble of any type and temperature, but this might also give reason for one to invest in FO drones in addition to Mirror Drones if making a hole in said Sheath is too difficult to allow the onboard laser weaponry to fire back. Probably would be best if there's a way to have "sectionized" Plasma Walls that form a kind of bubble instead of having an overall Plasma Sheath.

Either that or simply invest in Planar Plasma Walls that can only be projected and angled at specific sides of the spacecraft and aim the walls at the opposing spacecraft. I'm not a hundred percent certain, but I think that is how Honorverse spacecraft are protected, so correct me if I'm wrong.

- Sabersonic

Anonymous said...

"Re: plasmas. A plasma can absorb light with a frequency lower than the so called plasma frequency of the plasma. The plasma frequency depends only on the density of free electrons. I go into more detail here
http://www.panoptesv.com/SciFi/Ionization.html
but in brief, singly ionized air will not have enough free electrons to stop most weapon frequencies (such as 1 micron near IR, or anything in the visible, UV, or x-ray). Since getting any fluid with the density of air and the temperature necessary to ionize it around, this does not bode well for using plasmas to defend a spacecraft (in addition, if it did block laser beams, it would blind the spacecraft since light could not get through. It would probably also cook the spacecraft from the radiated heat)."

I've read about recent research on 'cold plasmas' that are only a few hundred degrees and close to atmospheric densities...there are three ways to create plasma; heat, radiation, and voltage. 'Cold plasma' uses voltage to both generate the ionization needed as well as confining it. I'd give you a link, but I'm pressed for time...gotta go to the dentist. A plasma sheath would act like ablative armor, but you could renew it (until you ran out of feedstock for the thing)...thus acting more like a 'force sheild' only more realisticly.

P.S. I should read ALL the post before I submit my own replies!

Ferrell

Rick said...

Jean - On 'shooting from the ranch house windows,' the reference is to Western movies, with guys inside a house moving from window to window under cover, then popping up to take a shot before they can be targeted.

The idea is that uber-range and jumbo mirrors could be irrelevant if lasers have to take quick shots through armored shutters, with the optics perhaps on a track arrangement so they can fire through multiple shutters.


Replying to a whole subthread here: On heat sinks, I'll note Ken Burnside's elegant surrender arrangement in his game Attack Vector, where ships 'strike their colors' by extending their main radiators.

So far as I can see, if the laser itself is buried inside the hull, having multiple lasers and multiple heat sinks provides redundancy but not additional power - your zap capacity and heat sink capacity is determined by the total, not how it is subdivided.

But one other issue with internal lasers (in fact, uber-powerful lasers in general). So far as I know, there are no perfect mirrors or lenses that absorb none of the incident light photons. Keeping your optics from overheating and degrading might become a serious problem for extremely powerful lasers.


Mirror drones - Note that my objection to 'fighters' is mainly to piloted craft in these roles, which don't seem to call for high level on-spot tactical judgment (and are likely to suffer severe attrition). Drones aren't a problem.

But a constraint on mirror drones could be the cost of big mirrors, equivalent to observatory telescope mirrors. If mirrors are cheap compared to the laser itself, mirror drones have big advantages, but if big mirrors (or other optics) are a substantial fraction of base laser cost, you'll be limited in how many mirror drones you can provide, and they become high value targets in their own right.

Citizen Joe said...

If you've got an hour or two, check out Journey to Palomar available on PBS and Hulu:
http://www.hulu.com/watch/76165/pbs-specials-journey-to-palomar

The particularly relevant stuff about fabricating mirrors are probably in the last half of the show.

And yes, the mirrors are expensive.

Rick said...

Citizen Joe - Embarrassing confession that I grew up in San Diego, but never visited Palomar.

Presumably the cost of a given size mirror, such as 200 inch = ~5 m in Palomar's case, has come down and will continue to, so that the current state of the art is 10 m. But (also, presumably) whatever rates as a Really Big Mirror will be expensive - expensive enough that you want to mount it on your main laser platform.

Which doesn't preclude smaller mirror drones in both offensive and defensive (escort) roles.


Sabersonic - But the big mirror segues back to this point of yours: Me thinks that this might make an excuse to have a kind of Fighter Pilot mentality for the helmsmen of such spacecraft in addition to a kind of Top Gun Orbital Combat School, but that's just me.

The skill set would be very different, but I can see a case for a huge battlestar being handflown, in some sense, in combat - precisely because it is so cumbersome that you may have to think three moves ahead, so to speak, to get and stay on target.

At Stupendous Range it shouldn't matter, since angular velocities are low even for high speeds, but cluttered space could be a different matter - and I'm more and more inclined to think that cluttered space is where things get complicated, and interesting.

Anonymous said...

After watching the Fifth Episode of the History Channel series "That's Impossible" on Directed Energy Weapons and the like, a though occurred in my mind: Since these combat spacecraft (and eventual starcrafts) are heavily reliant on electricity and power, what's to stop the development of E-Bomb warheads that create a localized EMP that disables all the onboard electronics?

It doesn't even have to be mounted on a missile platform, just carried by a small shuttlecraft whose electrical components and circuitry are hardened and protected by the effects of EMP to get close enough to use the E-bomb warhead. Might even preclude a boarding party onto the combat spacecraft.

Just a simple brain fart. There are probably circumstances and limitations to the use of such a weapon. Though it is an interesting scenario, not really sure if this is ideal for comandeering spacecraft since I have no idea as to how easy (or difficult) to repair EMP effected electronics.

- Sabersonic

Jean-Remy said...

Saber:

Two words. Faraday Cage.

In other words, isolate all your electronics and computers inside a structure such that any electric potential will course along the surface without penetrating. In fact, the ship itself, depending on construction materials, might well be one.

Consider a jetliner and a car as Faraday Cages. Either can be repeatedly struck by lightning without it affecting their electronics. Basically take any conductive metal enclosure and isolate it. Electric potential should remain on the outside of the enclosure. It need not be slid: a wire mesh should be sufficient depending the circumstances.

I am sure there would be other tricks to harden your electronics against an EMP. If EMP missiles (nuclear or not) are used, I can see them as more effective against civilian ships: in order to save mass and/or construction costs, their electronics might be more vulnerable, and it would perhaps make a good non-destructive way to disable them, either by border patrols or pirates. If the pirate problem is bad enough, perhaps even commercial vessels, at least those operating in dangerous zones, would have protected electronics.

Citizen Joe said...

EMP is not going to be effective against space ships because spaceships are constantly bombarded by radiation. Thus, the ship's electronics are necessarily protected.

H said...

I like the idea of those mirror drones, however would these not be limited by the same constrains of a turret based laser?
After all the mirror you can carry on board a drone would be of limited size.

If turrets suppose too many limits for lasers, I imagine there could be need for multiple emitters pointing to different directions?

I also see a problem with gigantic lasers: the bigger the "cannon" the less surface it has in relation to its volume in order to expel heat. Could there be a limit in how big these systems can get?

Luke said...

Re: cold plasmas. I still don't see how you can get enough electron density in one of these to absorb a laser, especially when trying to confine it against escape into the vacuum of space. One note, though - at frequencies above the plasma frequency for its electron density, the plasma is transparent and dark. Below the plasma frequency, the plasma is opaque and radiates like a black body of its electron temperature, and the electron temperature of a cold plasma is still about 10,000 K even if the ion temperature is near room temperature. If the electron density is high enough to stop near infrared or visible laser beams, the effect on the "protected" spacecraft will be like being in a broiler oven, and the power drain will be tremendous.

Re: EMP. EMP tends to be quirky and unreliable, even on unshielded systems. Those devices that are damaged by EMP will have electronic components burnt out - repairing basically means replacing the damaged components with spares. Some systems will be inherently vulnerable, mostly those that need to pick up or transmit radio and microwave signals such as radar systems or radio communications. However, there is no guarantee that future computing systems will be vulnerable to EMP - optical or spintronic computers would pretty much ignore it, and the nanoscale carbon computers people are talking about as The Next Big Thing will be far more resistant to EMP than today's silicon electronics. Silicon computers can be shielded from EMP by putting them in specially designed conductive housings and using what are essentially fancy surge protectors on any incoming conductive cables that might act as antennae(they need to operate on a faster time scale than home surge protectors). Note that radiation hardening is entirely different from protecting versus EMP.

Re: mirror drones. A mirror drone could be larger than a turret. The drone could use a segmented mirror. It would be stored with the mirror folded. After launch, the mirror would deploy to many times the diameter of the stowed drone.

Anonymous said...

Heat removal from the laser would be scalable up to an extent; when the expense of operating the cooling system becomes equal to that of the laser generator should determine the upper limit to both.
As to cold plasma; I'm sure that the scientists working on it have taken the heating and optical windows into account when they design the specific applications for plasma protection systems. Plasmas are strongly affected by both magnetic and electric fields; confining them through voltage is probably a matter of overcoming engineering hurtels, not fundimental physics. I'm sure practical solutions to these problems will be made as the spacecraft evolve.

Ferrell

Rick said...

I'm just sitting back enjoying and absorbing this discussion without really having anything to add to it!

But I would think that optically thick dust would be effective as a smokescreen - sure, a weapon grade laser could zap through it quickly ... but where? Like a traditional naval smokescreen it doesn't block fire, but it obscures the target.

Of course, the smokescreened craft can't fire either, at least not with lasers.

Jean-Remy said...

Rick: a mild concern about the dust screen:

It works very well as long as you're not doing any delta-v since the released dust will exit the craft on the same vector. The moment you need to apply accel. (and if you're in combat that's probably something you have in mind) You'll be getting out of your dust cloud pretty quick, requiring a constant stream of dust. This brings me to the next point: how much mass and how much volume would a sufficient amount of dust to mask the ship in combat take? Also, how do you spread it out evenly around your ship? A magnetic field perhaps would even out charged dust, but then not only can't you shoot, you can't even see through your dust screen. And while your enemy doesn't know where you are in there, he does know you're in there, somewhere.

Also, what would you use to shoot through? Lasers are out, probably rail and coilguns would (?) be out to because it would affect the magnetic field, and the exhaust plume for a missile is sure to scatter the dust.

Note that these concerns fit the plasma screen as well. At best it puts the two adversaries on equal footing, and any tactic that does not confer an advantage is useless. At worst you handicap yourself more.

Z said...

I have to second the notions of cluttered space being where things get interesting. I would add a corollary and say that interesting space might be the only place where interesting things happen- space opera tends to have battles set in deep space, but what the devil is out there to fight about, resupply you, or hide behind? No one is especially keen to show up to formal duels with sniper scopes when they can wait in the alley with three of their friends to shoot you-and you are welcome to attempt the same.

On a more technical note, I have one more laser defense notion off the top of my head. Over at Atomic Rockets, there is mention of rotating missiles being inherently harder to laser kill- but what if you took it one step further, and spun up your missile/ship/lancer by spinning a contrarotating shell (if you don't mind an internal radial acceleration, perhaps for artificial-g in a manned craft) or two of them (if you do mind.) Now you can spin the exterior of your ship at an arbitary rate, and to make a hole, you basically have to cut a ring in both (or all three) hulls. Fill the space with propellant, use impellers on the hulls to keep it flowing past, you might be able to pull a pretty arbitrary amount of heat away from the hull (this is of course in a universe sans Death Star-esque terawatt instakill gamma ray boomsticks.)

Just running with this (and assuming I haven't made a kindergarten error, which, given the local hour, is wholly conceivable) it would seem that in a roughly cylindrical ship, you now have a preferential attack angle- the ends of the cylinder, where the size of the ring described by the beam and the the hulls is smaller and thus can absorb less energy. More practically, it's probably where you need to put an engine at one end and your big laser at the other. You probably want to go for the engine, to avoid the various laser-on-laser difficulties.

Engine at one end, guns to the other, and you can only reliably kill them with the gun aimed at the engine...hmmm...;-) I'm fully onboard with space war as space war and not naval or air war in space, but I found it an interesting corollary all the same. Which means it is probably wrong.

Rick said...

Jean - Yeah, smokescreens have their limitations! OTOH, if you are on orbit toward where you want to go, it could be a way to protect yourself as you pass through a threat zone.

The dust might not have to be magnetically contained; after all, it will only dissipate at the speed at which it was ejected. I don't have any idea how thick it has to be, just 'optically thick.' I'd guess that means enough dust that a particle blocks every line of sight.

Zeroth approximation, if dust particles are a micron across and separated by 0.1 millimeter, the dust would have to be several tens of meters deep, with mass on order of 1 gram/m3, or 1 ton for a cube 100 meters on a side. Thus about 20 tons for a 'pancake' of dust 100 meters deep and 500 meters across.

Beams fired into the dust will vaporize the grains they hit, and the expanding dust will push nearby grains away, but not very quickly - the gas from an exploded grain is pretty damn thin by the time it hits neighboring grains.

(Better-informed observations welcome!)


Z - Well, the gun (or rather the main mirror) is another good target, which gets into the eyeball frying contest.

Rick said...

Z - Forgot to add that I agree fully with the premise that space engagements won't typically be in the middle of nowhere, except perhaps when a task force on orbit from A to B is intercepted by a defending force dispatched from C.

The real question, in my mind, is whether engagements happen in the orbital clutter, or a ways beyond it, as would happen if the defender takes a long ellipse to intercept the attacker at apoapsis.

Z said...

I have questions about the possibility of engagements the other direction, altitude-wise- transatmospheric shootouts. I could see substantial use of aerobraking/gathering air for reaction mass or fuel depending on the drive/gassing up at airborne tankers/deorbiting and hiding half an orbit in the clouds to shake a persuer all having a place, presuming you have enough delta-v to orbit yourself a couple times. I remember talk of the X-20 changing orbital inclination by effectively flying, so someone has at least chewed on it a bit. Even suborbital craft might have a place-hop up high enough to get a clear laser shot over the horizon (and out of the air, depending on the wavelength of your laser), fall back down by the time they respond, gas up from the air or a tanker, repeat.

Citizen Joe said...

RE: Dust shield. Try a big honkin' block of ice in front of you. Ice is all over the place out past Mars and free for the taking. Ice is quite reflective, thus resistant to visible light lasers. Ice has an enormously high heat capacity. Ice, when vaporized will eventually reform into ice. Ice can also be harvested for reaction mass. A big old iceball could be held in front of your ship with a grapple arm. If you need to peak or hide, you use your grapple arm to maneuver yourself so that the ice is between you and harm. The beauty of lasers is that you can target vital components with pin point precision. Ice doesn't have vital components. The enemy might have the ability to destroy a crucial piece of equipment, but it probably can't pump enough joules into your ice shield to completely vaporize it.

RE: Mirsiles. Spies Like Us had that sort of set up, with satellites set up to reflect a laser around the world. On the one hand, each mirror in the system will absorb some of the laser energy because it is not a perfect reflector. On the other hand, the mirrors can re-focus the laser, extending its range that way. You can even use your own laser as a communicator to tell the satellite to prepare for incoming high power laser.

Rick said...

Z - In my first Space Warfare post I talked quite a bit about exatmospheric pop-ups, but in the context of missiles.

Given conditions of laser dominance, laser aerospace craft are certainly also a possibility. Mounting a Really Big Mirror would probably require that it be segmented (which may be desirable anyway to limit fracturing, as Anthony Jackson has noted at SFConsim-l), and deployed after leaving effective atmosphere (which adds some difficulty).

In the near and midfuture, at least, laser aerospace planes would probably use chemfuel to power the laser (not necessarily chemical lasers; perhaps turbogenerators). Limited zap duration is not an issue, since their missions are brief anyway. Go up, zap, return and refuel, rinse and repeat.

I'm rather skeptical of atmosphere scooping save in a far future tech, because of the enormous heat load involved.


Citizen Joe - That ice block isn't a smokescreen, it is expendable armor. And like any armor thick enough to be effective, it would need to be very heavy!

Citizen Joe said...

Unlike armor, the ice block is already in space and free for the taking. Also the ice block can be used for remass and the armor can't.

Rick said...

Depending on where you are in the Solar System. Around the outer planets ice is cheaper than dirt. In Earth orbital space, not so much.

Luke said...

RE: Dust. The big problem I see is the dust will tend to disperse. With the number of particles encountered by a ray equal to (cross sectional area of particle) * ((# of particles)/(volume of cloud)) * (path length through cloud), and the dimensions of the cloud increasing linearly with time (and thus the volume increasing as time to the third power), you see that the number of particles encountered by a ray goes down as 1/(time)^2. When the number of particles encountered by a ray becomes less than one, you are no longer hidden. Further, early on, the dust cloud is so small that the uncertainty of where you are is also small. Just for giggles, lets assume 1 ton of dust with a particle size of 1 micron, a particle density of 1 g/cm^3, and dispersing at 1 cm/s. This gives a cross sectional area of about 1E-12 m^2, 1E18 particles, a volume of about 1E-6 m^3/s^3 * time^3, and a path length of about 1E-2 m/s * time. Solving for time when the average number of particles encountered equals 1, we find that this shield will last for about 30 hours, and will become essentially transparent when it is about 1 km across. It seems like this may be operationally useful in some circumstances - namely when you do not need to maneuver and when you have plenty of time to deploy the screen. Similar screening possibilities might include thin aluminumized Mylar balloons inflated with a diffuse gas (you don't need much when the exterior is vacuum). These will not disperse with time and may be able to be deployed significantly more rapidly.

Re: Cold Plasmas. Plasmas are not confined by voltage nor affected much by electric fields, since they are overall electrically neutral. They can be confined by magnetic fields so long as the energy density in the field is higher than the energy density in the plasma (otherwise the plasma expands, taking the field with it). At atmospheric density and temperature, the energy density is 0.15 MJ/m^3, and thus requires considerable energy stored in a magnetic field to confine over any significant volume - and even this has insufficient electron density to stop near IR and visible lasers. Further, I don't see much informed talk about using cold plasmas to defend against lasers - the people who know what they are talking about mostly refer to shielding versus radar and microwaves, which requires a far smaller electron density.

Re: Mirsles. Dielectric mirrors can be matched to the wavelength of the laser to be reflected, leading to reflectivities of 99.99%. With these reflectivities, loss of power from reflection is negligible. Some additional power may be lost due to the mirror not intercepting the full beam, but for a single reflection this can be manageable and you could still deliver more than 90% of the power of the primary beam to a target at much larger ranges than possible for the focusing elements on the spacecraft itself.

Anonymous said...

Luke- all true:
"...and even this has insufficient electron density to stop near IR and visible lasers. Further, I don't see much informed talk about using cold plasmas to defend against lasers - the people who know what they are talking about mostly refer to shielding versus radar and microwaves, which requires a far smaller electron density."
However, electron densities aren't the only property of plasma- the positive ions that make up the rest of the plasma have an effect on EMR. The chimecal makeup of the plasma also has an effect, although I'm not sure to what extent.

Oh, and dust clouds also have the property of defracting (bending) beams of EMR depending on the frequency and particle size. Scattering or defocusing a laser beam, even just a little might be the difference between causing your hull to glow cheery-red or have a hole burned through it.

Ferrell

Jean-Remy said...

Mir-sles: Russian Peace-Missile

(Mir being peace--or world--in Russian)

Sorry the oxymoron just struck me as hilarious, I had to throw it in here. You may return to your regularly scheduled seriousness. (I was going to say geekiness but then I realized that a pun involving knowledge of a foreign language and a funny alt spelling of missiles was geeky in itself)

Rick said...

Luke - a 30 hour lifetime for the smokescreen would be tactically significant! (And could be replenished from the back.) That said, balloons have the advantages you mentioned. One possible limitation to balloons is that slicing through it with the laser might give a glimpse through it, an aid in localizing the ship, whereas turbulent motion in dust might tend to fill holes zapped through it.

Citizen Joe said...

Let me go off on a tangent and then bring it back around to lasers.

In the game Traveller, they have meson cannons which accelerate mesons to a specific fraction of C such that the time dilation and predictable decay rate can dictate when they decay and thus explode. Prior to that, they can't be detected and pass through all matter. The defense is the meson screen. Anyway, one nasty orbital defense technology is to put meson cannons inside subs (like ballistic missiles) and the water depth prevents their location. They of course need some sort of forward observer. The counter to that was the supertanker lab ship. That vehicle had a very large tank of water. When the mesons decayed inside the water tank, the disruption in the water could be detected and a targeting solution could be calculated for a return salvo.

Going back to lasers. The first sign that you're being attacked by a laser is a gaping hole in your bulkhead. You can't normally see a laser at all, especially not a non-visible light laser. Returning an attack from a sniper could take many hits before the right location is found. A dust cloud, however, would leave a tell tale vector as the laser illuminates the dust. Stopping the laser is less important than giving away the enemy's location.

Luke said...

Lasers don't put all of their light into the central spot - this is an approximation, but some leaks out. I've got some pictures of the beam profile at the focal point at http://panoptesv.com/SciFi/Diffraction.html
which show the way you get ringing and bleed-out beyond the area being subject to melting and/or evaporation. Sensors near (but not on) the spot being illuminated should see the beam's light, and be able to track back to where it came from.

Re: Traveler meson beams. Ugh. Talk about twisting known physics into a pretzel. They are wrong in so many ways.

Anonymous said...

After reading these last few posts about ways to defeat or at least lessen the effects of laser bombardment, a thought just occurred in my mind.

Okay, I admit, it's more of a brain fart. If there is such issues with the dispersion of the dust field or any similarly used laser diffusion effect in free space for a defending ship while it moved, then why should it be deployed that close to a defending spacecraft?

From what I've read in the Space Fighter, Not blog entry, at best the targeting data for laser weaponry to reach the attacking spacecraft if and when both spacecrafts are mere light-seconds from each other. Meaning that a defending spacecraft has about six seconds to maneuver and avoid being targeted and its vital systems shot and disabled. However, this time could also be utilized by launching a "Laser Diffusion Field" missile in between the two crafts, normally closer to the attacking spacecraft than the defending spacecraft, that decreases the overall damage potential of the laser beam/pulse and the damage the defending spacecraft receives.

Unless I'm wrong, the Laser Diffusion Field would more or less move in the same direction and velocity as the two spacecraft the moment it is deployed. Naturally, the attacking spacecraft will try to maneuver itself around the field to bring its onboard laser batteries at full "power", for lack of a better word, against the defending spacecraft.

The defending spacecraft, meanwhile, is attempting to keep the field in between itself and the attacker, yet trying not to remain in parallel with the field since it would not keep up its strength indefinitely and an off-axis shot is far more preferable than a direct shot.

Granted, this defense idea works best at higher, less cluttered orbits than lower and more populated orbits.

- Sabersonic

Rick said...

Citizen Joe - You don't see a laser beam coming before it zaps you ... but you see the firing ship, and if you know or suspect it is a hostile you're already taking defensive action - including returning fire, in whatever form.

Though this also relates to the whole question of how different things get if you're fighting in an orbital clutter and can't readily distinguish hostiles from the clutter till they do something hostile ... like zapping you.



Luke - LOL on your comment about meson beams!


Sabersonic - The hitch is that, short of really uber performance, a few seconds won't allow any defensive screen to get very far from the craft it is protecting.

Citizen Joe said...

That brings up the question of how you know you've been hit and where? Do you mount cameras outside? Do you set up a sensor network through the entire hull? Do you just measure pressure drops?

And, assuming you've noticed the hit and found the hole, which one of those many dots out there do you think is the attacker? What if the attacker was occluded by a planet and/or star?

Citizen Joe said...

What is our realistic estimate on effective range with lasers? Given the need to get them into orbit, the huge free electron lasers and the complex Chandra array would probably put Xrasers out of contention. UV lasers are more likely. And what are our capabilities to make a focusing mirror for that laser (and getting it into space). It is one thing to formulate an hypothetical 100 meter mirror, it is quite another to actually fabricate it.

Rick said...

Citizen Joe - You pretty much know you've been hit because something critical has been zapped, or the armor protecting it has. Lasers don't take random shots; their aim is on the same order as effective spot size.

Realistic for what era? :-) For the midfuture, I'd expect to see lasers in the near and mid-UV, maybe 200-400 nm, with mirrors upwards of 10 meters. That's the current maximum, and probably costs on order of $1 billion, not huge for military budgets.

I'd imagine that by next century it would be practical to build 25-meter mirrors, maybe 50-meter; 100 meters is so far past current practice that it is hard to say whether it will be practical with midfuture tech.

That would give an effective range of several thousand km. You can ask similar questions about beam power and how long you can hold down the trigger.

Jean-Remy said...

On range, I would say that once you can get a laser that delivers enough energy do start doing damage at half to three-quarters a light second away (that's including diffraction) you've pretty much got it where you want. Anything beyond a light second range and all targeting data, by the time you get your shot off, could well be invalid: at best there's a two-second delay between a firing solution and the delivery. If your enemy is going at even the "low" speed of 10 m/s your shot could be off by as much as 20 meters, hardly the sniper shot to the enemy's optics you want. (Granted if the ship is coming straight at you it won't change anything, or if the angle is steep enough the difference could be as little as a few centimeters. Twenty meters is the worst case scenario with perpendicular vectors.)

I still suspect that any delay significantly longer than that will start to make pinpoint accuracy somewhat of a concern, so getting more power to combat diffraction to get a longer range becomes a game of maximization over optimization, ie: Law of Diminishing Returns apply.

Citizen Joe said...

I don't think I agree with the notion that you can look through the same lens/mirror for targeting that you're using for firing. Yes, a visible light laser would work because all visible light is a really narrow band in the spectrum. However, targeting is typically done in the IR range or even microwaves. Meanwhile the damage is done in the UV range. The materials that handle those two frequencies are quite different and the focus for each is significantly different.

Jean-Remy said...

However wouldn't that mean having not one, but TWO 10-20-50-100m mirrors mounted on the ship, to have the same resolution as your laser and precisely target a subsystem rather than "sort of shooting in the approximate direction of the starboard radiator"?

Luke said...

Jean Remy: If the target is several light seconds distant, but traveling on an unpowered path, you still hit because you know where he will be when your beam arrives. The difficulty occurs when your target accelerates. However, this can be used to your advantage. As mentioned upthread, if you can force your target to keep accelerating so as to avoid being damaged by your laser, your target is wasting precious propellant. Once the target no longer has enough propellant to complete its mission, you win.

Citizen Joe: If you have a UV laser, you will can your final targeting with UV light, so as to provide an image in your beam pointer scope that you can put the cross-hairs on. Even if there is not a lot of ambient UV light around to illuminate your target, this is no worry - you have a very bright source of UV light with you, in exactly the wavelength band that can be handled by your optics - the laser you intend to use for shooting. Use it as a flashlight for the final targeting solution (or, more accurately, scan the beam back and fort to illuminate the target). In x-rays you might need to get a bit fancier - you would scan the beam and detect the x-ray fluorescence radiation. Since this is at lower wavelengths from your beam, you would use the fluorescence telescopes to tell you where your beam is pointing when it is on target - then put your beam back in that position and turn it on full power.

And in the very near future, we have put 3 meter telescopes into orbit, typical laser wavelengths for battle beams are around 1 micron, and you might expect megawatt powers. If you focus a 1 MW beam into a 10 cm circle, you drill through steel at a rate of around 5 cm/s (the beam's heat first melts the steel, then vaporizes part of it and the pressure from the iron vapor jet blows the melt out of the hole). The aforementioned setup of 3 meter mirror and 1 MW 1 micron laser can focus into a 10 cm spot at a range of 300 km. At 3000 km, the spot size is 1 meter across and the temperature is too low for vaporization. The melt front will propagate into steel at a rate of 0.2 mm/s, so you will need fairly long illumination times at this range (seconds to minutes).

Citizen Joe said...

If you 'paint' your target with a low power beam, then that targeting laser is going to give away your position as it sweeps a sensor and it will also warn the target that it is about to be attacked. A passive system is needed for first strike capability.

Three hundred kilometers may seem like a lot planetside, but in space that is almost on top of each other.

Luke said...

Active illumination is similar to using radar for target acquisition. It may give away your intent before you shoot, but is still used because it is a useful means of guiding your weapon. Likewise, your target no doubt already knows you are there, although it may not know you are hostile. Using active scanning at short ranges (where dodging is not possible) will make no difference. Active scanning at multi-light second ranges may be unavoidable (particularly for x-ray beams). After shooting, your target will know where you are anyway (his sensors that are not blinded will have seen a bright point of light coming from your position - they will be close enough that they can detect the spillover from the beam that does not hit the focal point).

As far as ranges, 300 km is quite respectable for point defense. In NEO, with relative velocities on the order of 7 km/s, this gives you close to a minute in which the incoming kinetics can be ablated away by your laser at a high rate (a rate which only gets higher as the kinetics get closer). Meanwhile, 3000 km is the distance to the horizon from orbits at an altitude of 13400 km, or most of LEO. This makes the laser a useful orbital offensive weapon since the time it takes an enemy to cross your point of view from when it first pops above the horizon to when it sinks below the horizon again will be on the order of 7 minutes. In seven minutes, that MW near infrared laser can melt out a 9 cm deep hole in steel 1 meter across. Since near future spacecraft will be very lightly constructed, this will be quite effective at disabling them.

Jean-Remy said...

Luke:

I'm not sure I would count on my opponent using all his delta-v reserves in order to dodge me. In fact I'm pretty certain that dodging incoming fire was, in fact, calculated in the delta-v reqs for their mission.

The P-51 Mustang was developed as a long range bomber-escort to replace previous fighters for that very issue. The P-40 Tomahawk had enough range to escort the bombers but not enough fuel left to actually do any fighting when they got there, leaving the bombers vulnerable. The P-51 had a longer range due to the addition of external fuel tanks, allowing them a given amount of fight time when they got to the objective. Operational range does not just include enough fuel to reach a destination and return, but had better include the fuel to do the actual fighting once at the target. And if the target is important enough to hit here and now, well there's always the panned return delta-v budget you can throw at it. The ship might not return, but if you can kill the Death Star before it becomes fully operational...

Of course it is always possible for the enemy to go over-budget, but even then he's probably keeping an eye on his fuel gauge, and he's going to try and disengage well before he hits "bingo fuel", so I would not count on them surrendering to you because of fuel consumption. Unless they really really really messed up at Intelligence or Mission Planning.

Citizen Joe said...

That is why I think "The Cold Equations" is full of bunk.

Passive sensors are going to be effective at probably a hundred times the damaging range of lasers. Scanning range is probably ten times the damage range, with 'blinders' operating at the same range.

So a month before contact, ships will know the others are out there. This equivalent to blowing fog horns. You know they are out there, but not well enough for targeting. A couple days before contact, they can scan each other and/or blind each other with countermeasures. Finally, an hour before contact, they can actually damage each other but not precisely. Maybe firing into the countermeasure beam.

This is really starting to look like the battle against the medusa where you can't look directly at your target. It also lends credence to the many small fighters operating at scanner/countermeasure ranges and feeding data to the main gun.

What would the policies be on scanning? Probably scanning beyond damage range is 'legal' but scanning inside damage range is considered aggressive.

Luke said...

Re: Scanning. Active scanning with hard x-rays is probably always fairly hostile. You are, after all, irradiating someone with ionizing radiation, and enough x-rays to produce noticeable fluorescence will likely damage even rad hardened equipment such as solar panels or electronics not buried under a significant thickness of heavy elements.

Scanning with UV, visible, or IR lidar is probably fine as long as it is below the threshold for damaging sensors. However, it does mean someone is taking an unusual interest in you, more than is needed for mere navigation. Miners and prospectors might use lidar on asteroids, surveyors on planets from orbit, but folks using lidar on other spacecraft are at least suspicious of you and possibly dangerously interested in you. Lidaring is probably a good way to ratchet up the tension level, even if it is legal and itself does no damage.

Re: Dodging vs. Delta V expenditure. An enemy may budget propellant for combat maneuvers, but past a certain point this becomes impractical. Consider a spacecraft 10 m long and wide at a distance of 5 light seconds. To dodge, it needs to change its position by 10 m in 10 seconds. This requires an acceleration of 0.2 m/s^2. The target will need to keep up this acceleration indefinitely, so it will spend 0.2 m/s of delta-V every second, 12 m/s every minute, 0.72 km/s every hour, and 17.3 km/s every day. 0.2 m/s^2 is too much for ion or other plasma thrusters, so you will need to use higher thrust but lower specific impulse drives like chemfuel or nuke thermal. With nuke thermal and a large propellant tank (with 95% of your spacecraft mass as propellant), you might get 30 km/s of delta-V, total. At typical interplanetary orbital speeds, say 10 km/s, it will take 40 hours to cross 5 light seconds, which works out to 30 km/s of delta-V. This will exhaust your ability to maneuver in a typical crossing. If you get closer than 5 light seconds, the rate of expending delta-V goes up.

At multi-light minute ranges, dodging could work for a while. At ranges of several light seconds, dodging just means you are living on borrowed time.

Citizen Joe said...

The dodging requiring delta V implies a single ship. If your ship is composed of micro-sections on maneuverable trusses, then you could reconfigure your ship so that nothing is where the laser is aiming. Sort of like an amoeba. Yes, that would be a fantastic feat of engineering and a nightmare for the repair crew, but if it is that or getting vaporized, I'll take a grumpy engineer.

Rick said...

75 comments!

I am paying the price now for not posting about 'the midfuture' and its plausible techs, because we are mixing techlevels here wildly.

Beams capable of zapping at light second distances imply a pretty high techlevel; either really big mirrors or X-ray lasers. If you have lasers like that you may also have torch level drives that can sustain evasive maneuvers.

(But note that these jinks are really not very tactically interesting; they are about dodging beams, not changing the battle geometry.)

There is also a question of how long you can hold the trigger back. The power supply may hold out, at this techlevel, but you still have to get rid of up to gigawatts of waste heat.

In the midfuture I expect laser ranges to be much more modest, but still useful for point defense against incoming kinetics. (And enemy spacecraft if they come in range.)

Passive sensing should be enough for most tactical scanning and fire control - but once the fight is on, you're not giving anything away by pinging.

Citizen Joe said...

Passive sensing (particularly IR) is feasible for targeting a ship as a whole, but you're not going to get the pinpoint resolution, extolled by Rick, which allows targeting critical systems only. Target discrimination requires painting the target with an appropriate radiation. A scanner capable of sensing the fine details is going to be fried by the direct application of that same radiation. This is why it is very hard to see a ship with the sun behind it.

Navigationally speaking, passive sensors should be fine for moving around and you can learn a lot about another ship from sensors alone. However, you can't target subsystems with that.

Again that brings you into four range bands. Farthest away is the one where you can't discriminate a ship from other radiation sources like stars. Then you've got your passive band where you can track a ship and know how much radiation and type it is emitting. Then you've got your scanner range, which relies on active pinging or painting of the target. And the final range band is where you can effectively shoot them with your lasers.

As two ships close to firing range, I think the policy would be to scan first for intention and then blind each other if intent is not specified. If the inbound vessel was unconfirmed but signaling non-hostile, then the system ship would scan/blind it and take control of the inbound vessel. This is basically a harbor pilot situation.

Of course, that model assumes similar technologies. If you slave a FEL Xraser to your UV Scanner, then you're likely in firing range as soon as you can scan. I suspect there are weird issues with that as well, regarding differing focal points, but it makes for a very ugly surprise for ships thinking that they are in the friendly scanning range still.

Jean-Remy said...

I'd think a way to approach another vessel would be actually very similar to the Minbari tradition of greeting with open gunports. I think that is actually a very sensible and intelligent way to make a careful contact.

If you make your approach with all your sensing equipment extended, and therefore vulnerable, you show non-hostile intent. You're inviting to be blinded. If your weapons are lasers, same: Your optics are out and vulnerable to first strike.

If you approach with your ship buttoned up tight, and ready for a fight, that would be enough to paint you as a hostile.

Of course you could always approach with *most* of your systems vulnerable, but keep the secret extra mount well hidden and strike without warning.

Just because there's no stealth in space doesn't mean you can't be sneaky.

Rick said...

Citizen Joe - Last point first, if there's a major techlevel disparity you expect a one sided battle in any case.

'Whatever happens, we have got
the Maxim gun, and they have not.'


The whole blinding thing complicates my assumptions about passive fire control ... but it complicates active fire control as well. Any sensor able to detect fine detail, whether passively or from pings, is vulnerable to dazzling/blinding.


Jean - A whole fairly elaborate etiquette could develop for encounters of spacecraft in uncertain circumstances, determined by both the situation and the underlying technology.

Jean-Remy said...

Resurrecting an old discussion on laser weapons and tactics because I just had an idea.

So far we've pretty much decided that in a laser contest, they "eyeball frying" seems to be the most likely tactic.

However, what if instead of coming at each other head-on, with their mirrors pointed at each other defying to other to scorch it (and having it scorched) the approach is done while coasting in a "tumbling pigeon" maneuver.

The ship's computer is programmed with a pseudo-random series of attitude changes, making the ship tumble in a manner that cannot be predicted by the enemy, but know to the computer. Since the ship is in constant movement, it is nearly impossible to focus a laser on a single point long enough to melt through. It also becomes hard to predict when the other ship's mirror will come out of occlusion, so you can't "hold your shot" waiting for the mirror to appear. The tumbling ship however, knows precisely when and how long its mirror will face the enemy, and can time its fire. That is, unless both ships are tumbling. It is not impossible to predict when the enemy's mirror is going to be visible, and even more impossible to time the reveal of your mirror with theirs. You are left with taking picosecond potshots at whatever part of the other ship you can. Since most of the ship would be armored, you're more than likely going to carve some burn marks across the enemy ship armor, until, that is, you are at such range that even a picosecond shot with carve out the other ship's armor and find critical systems. Repeat until one of the two ships has lost enough critical systems to be considered "dead". If you get lucky, you might kill its mirror, but don't count on it.

Discuss.

Byron said...

I see, based on what is said here, that combat between laser-armed ships to be akin to that between Langston Field-equipped ships, in that in both cases seeing comes down to damage control. The sensors are easily burned out and have to be replaced regularly.
Also, I don't honestly believe that early lasers will achieve nearly the ranges predicted. The following equation from Space Mission Analysis and Design is a better equation for flux, at least in my opinion:
F=PD2/(piR2[(1.22LQ)2+(JD)2]
F is flux, P is output power, D is mirror diameter, pi is pi, R is range, L is wavelength (lambda, but I don't have that key), Q is quality, J is jitter (in radians). SMAD says that typical quality for 1999 is between 1.5 and 3.
Personally, I think that we won't approach 1 (diffraction limited) quality or zero jitter for a long time. A laser may be a jet engine in a telescope eyepiece, but the telescope has to survive quite a lot including shocks and heat that would destroy an observatory mirror.
And on the tumbling pigeon idea, it makes sense if you're facing a laser-armed enemy only, but it would be a bad idea in a combined-arms situation.

Rick said...

Our laser discussions (here and elsewhere) do tend to leave out the niggly little details of maintaining sub-microradian alignments in an environment that is a LOT less placid than the Hubble.

Also, in the near future you probably won't have the Hollywood six-shooter endless ammo supply in space, since it requires being plugged in to a multimegawatt power supply.

Rick said...

I forgot to say welcome to the comment boards!

Troy said...

In regard to the dust comments...

If you make the dust out of some substance with a high refractive index and low transparency, such as Titanium Oxide, it's going to totally scatter the beam without being fried. The individual particles are so small, their surface area to volume ratios are so high that all the heat energy is simply radiated away very rapidly. If you make your particle a dielectric, and its diameter one quarter the wavelength of the beam, it will reflect it pretty neatly (~99.99+% efficiency). Of course, this applies to mirrors, I'm not sure if they apply to individual particles. But you can bet this is what they'll use on the main laser mirror, because that's what people use today.

Turbo10k said...

About Interstellar/Deep interplanetary combat:

To travel between stars, or even between the inner and outer solar system, a Constellation would have to accelerate for months, even years.
I read a comment earlier on saying that the only situation where lasers can be used freely, in 'Blue water', would be in the case where constellation X intercepts constellation Y heading from A to B. The space-going equivalent of a clear horizon.

I do however have objections. Because on the long time spent accelerating on the part of X, constellation could only close relative velocities if:
-It accelerated at 1000g
-It is travelling at the same/similar speeds but at a different vector.

Option 1 is out, option 2 is unlikely.

My point is, there is no way lasers would be used in that level of clarity. 1 light second damage range, including lag? How much time would X need to cross that distance going at a nice 12%c?

Otherwise the interceptors would try to line up with constellation X to reduce arc distance, or just use kinetics (can accelerate much faster) or even better, mines (Kirklin or any large area, dumb KE mine bus). Lasers at perpendicular vectors would have engagement, damage dealing times of milliseconds! 1MW, 1 micron UV lasers with midfuture tech mirrors wouldn't even shave a millimiter off the other ships' armor!!!

PS: Can be summed up to lasers being interplanetary and 'brown water' weapons.

Rick said...

Welcome to the discussion threads!

I'll quibble that if you have ship speeds measurable as a percentage of c, you probably have uber powerful lasers that can deliver awesome zaps at stupendous ranges.

But I agree with your broader point, except to say that battles probably won't be fought in the middle of nowhere anyway, just as even 'blue water' naval battles were generally fairly close to land.

See the current discussion thread about Running on Rails, which touches on the threshold of local space in military terms.

I planned a longer comment, but I think I'll put further discussion on the front page instead - look for it there soon.

Anonymous said...

As in the "stealth reconsidered thread" most of the comments here quote theoretical laser performance and leave off a lot of the engineering troubles.

Some of the issues which were mentioned but need to be reiterated:
jitter
heat rejection (dumping waste heat)
combined arms (your opponent won't just be shooting lasers at you)
smaller ships are harder to hit than big ship and not just based on size but also maneuverability
finally lasers as PD would be great for killing the sensors on a KEW *BUT* do nothing about the mass behind that sensor!

If your opponent seeds your trajectory with grapeshot with no seeker heads, your 100 TW graser isn't going to do much for you (well a 100 TW laser fired in along your expected trajectory ought to vaporize everything in its path and cause it to dissipate quite quickly but you get my drift.)!

In an amusing way, this could recycle the whole concept of loading a black powder cannon with chains and grapeshot (only your cannon would be replaced by a coilgun or similar and grapeshot would be carefully calculated to be big enough to do severe damage.

Also I'd expect laser damage to be inflicted at a much lower rate than shown by the theoretical calculations shown here.

Consider reflectance (melted aluminum is VERY shining in most frequencies around optical), conductance, obscuration (by the heated plasma coming off of the target as long as the ship isn't under heave acceleration), jitter moving the beam around, etc.

Some of these effects make lasers better in space (no blooming and heat transfer to the atmosphere) and others hurt (no atmosphere to clear away the heated plasma of the target structure).

Jim

Anonymous said...

Another quick thought: "Telescope active optics."

Your laser weapon beam makes the ultimate "guide star" for active optics. You might ask, "why do we need active optics when we're not firing through an atmosphere?"

The answer is that your laser weapon and optics are going to heat - a lot. Active optics will enable your weapon to adjust its beam on the fly to keep the beam as concentrated as possible. Of course at very long ranges, I'm not sure how well the adaptive optics would work since the lag time is so great.

Anybody here work with adaptive optics? Is a 1 (or 2 or 60) second lag too great to be of any use?

Jim

Anonymous said...

There are 4 characteristics of a laser that must be considered in determine whether/how a laser will damage a target (I'll not discuss type [CW/RP] and frequency).

Intensity
Fluence
Power
Energy

Intensity is a measure of the instantaneous brightness the laser is at its brightest point. (e.g. E/cm^2/s). Low intensity means that the ship is too far away for the given laser weapon

Fluence is a measure of the accumulated energy at its brightest point (e.g. E/cm^2). A low fluence (with high intensity) may mean the laser can't fire or the beam director can't keep the beam on target long enough to deposit enough energy to damage the ship.

Power is a measure of the instantaneous beam energy integrated over the whole beam area (e.g. E/s). A low power beam with high intensity really should never happen. Focusing your com laser into a 1 nm spot on the enemy ship parked 2 meters away is likely to annoy the heck out them ;)

Energy is the energy deposited by the beam.

If the first three are high enough the damage inflicted by the beam ~ f(E). If the previous values are too low, than a very high energy beam may not be sufficient to do any damage (think of laser that couldn't do more than mimic the sun's illumination of the Space Station).

If one or more of the previous three values is too low, you get secondary effects which can help ameliorate the effects of the laser damage (conductance, reflectance, etc.).

Near term lasers may not have sufficiently high values in the first three values to get to the damage ~ f(E) realm. In the long-term I'm certain they will. So that leaves only the mid-term, which I suppose means it's up to the author :)

Jim

Turbo10k said...

Think of the difference between good quality, low scatter (or the other term for spreading out) but low power for communication lasers, and the punch-through your armor weaponized lasers....

Bookmarking.

Anonymous said...

Way late to the party, but had to offer a correction on pulsed vs CW lasers. Pulsed lasers are destructive at much, much greater ranges than CW lasers. If your pulses are short enough, you can easily deliver peak power in excess of 10^14 watts/m^2 across very large spot sizes. With this intensity, your laser's joules aren't heating the target, they're blowing it apart. Think of it as instead of putting a blowtorch to a steel plate, you stick a pack of C4 on it and hit the detonator. Midfuture tech isn't going to worry about terawatt CW lasers when you can use a pulse laser to deposit 5 gigajoules of energy in 100 nanosecond pulses spread out over a microsecond, which is the equivalent of putting a one-ton shaped charge on the hull of your ship and detonating it.

A 5 gigajoule capacitor can be charged in less than a minute from a 100 MW power plant. One ton of shaped charge per minute is pretty darned deadly.

Rick said...

Welcome to the discussion threads!

I've generally thought of blasting, as you describe, as more efficient than burning, but any real evaluation is above my pay grade.

Commenter Luke has a section of his website devoted to laser weapons, complete with sims of both pulsed and CW lasers. My impression from playing with them is that either approach is roughly comparable in all round zapping power, though some threshold conditions (and obviously details of laser and armor tech) might favor one or the other.

Anonymous said...

Yep, that's the same site that I've used, but the results aren't similar at all, so I don't know what numbers you were playing with. Blasting is far superior to burning, especially at distance and with lower power. Consider the results against titanium alloy for the following scenario:

One meter spot size.
5 gigajoules of energy.
For CW, 300 K initial temperature (very hot for space), 0 atmosphere (vacuum).

CW results: a 1 meter wide, .5 meter deep hole after one second. Jet speed is subsonic in the material, so no shock damage. Beam has to be maintained for one second.

Pulse delivered over 0.1 microseconds:1.59 meter diameter hole. Damaged diamater 2.46 meters. Hole depth is only 26 cm, but damage depth is 1.43 meters. Beam only needs to be maintained for 100 nanoseconds.

So, with a pulse, you get damage over 6 times the area and three times as deep. The pulse time is so short spinning won't help spread the energy out.

The results are even more dramatic with lower energies. With a 500 MW laser, a CW beam gives you a burn depth of 5.34 cm across 1 meter. The pulse gives you a hole depth of 14 centimeters over one meter, with damage over 1.4 meters to a depth of 68.4 cm.

Pulse lasers allow you to do more damage with lower energies. So, while you play your 5 GW laser on me for seconds at a time, I'll pound away at you with 5 500 MW lasers, doing 50 times the damage for half the power.

Anonymous said...

And just as a followup, my pulse lasers have a greater range than your cw laser.

Scenario: You have a 5 GW cw keel gun laser. I have 5 500 MW pulse lasers. We both have identical mirror sizes. As we close, I begin firing on you when my spot size is 10 meters. Each hit, I'm doing damage over ten meters to a depth of 54 cm (the hole depth is minuscule, the damage is all shock damage - think shrapnel). Meanwhile, in desperation, you fire you 5 GW CW keel gun at me. At a spot size of 10 meters, you burn off 4.5 millimeters of my armor every second. If I give my ship a slight spin, you'll burn off even less. If I have ten centimeters of armor, you need to hold that beam on one spot for 20 seconds to get a burn through. Meanwhile I've blown through five 10-meter chunks of your armor and sprayed shrapnel through your ship five times in one microsecond.

Rick said...

The comment flood on a couple of recent posts means I don't even really have time to analyze this scenario. But I probably wrote this post before seeing Luke's sim, so you can pretty safely trust that the sim trumps anything I wrote here.

Anonymous said...

His sim for pulse lasers does have an error; it fails to check for peak intensity. If that falls below 10^13 watts, your pulse doesn't do much of anything, so I had to figure those numbers manually assuming a nanosecond pulse duration over the spot size.

Rick said...

I thought it did check for that, but you caught me in a bad time (holidaze, etc.) for playing with computations and sim results.

Luuuuuuke!

Incidentally, I encourage posters logged in as 'anon' to sign a name to their posts. (It doesn't need to be real, just consistent!)

Byron said...

I have one more question about this topic, even though it's been dead for a while.
Why do we assume that the generators for CW and pulsed lasers are the same? Yes, it may be 1.5x as energy-efficient to use a given pulsed laser, but what if the pulsed generator is twice the mass and cost?
I would think that pulsed lasers would be bigger, simply because they have to handle a lot more energy at once. I'm not sure about this, and Luke may shoot me down, but I thought it needed to be said.

Anonymous said...

(SA Phil)

So what are the practical limitation on these Mirrors and Lenses?

Could your drones be Lenses or mirrors? Allowing your big GW Laserstar to effectively fire farther (smaller spot)?

Rick said...

Byron - Really the answer is above my pay grade, but my guess/understanding is that the pulse rate would be high, in the kHz or higher, so that the energy per pulse would not be that enormous.

SA Phil - Welcome to the comment threads, by the way! I don't know of an inherent reason why you couldn't mount a refocusing lens/mirror on a drone. But depending on propulsion tech (etc.), it might end up more cost effective and flexible to simply mount a laser on it as well.

Anonymous said...

(SA Phil)

Thanks for the welcome.
---
Purple vs Green:

So the problem from how I can see it is given lasers that are effctive out to 1/2 a light second or more, the fact they have "limited range" compared to kinetics is a non-starter given the actual likely relative closing speeds of any weapon that has a longer range than they do (such as a missile or other kinetic weapon)

Making the Laser the winner in the purple vs green (which one is which?)debate.

Unless you can make a suitably faster kinetic.

--------
Remote Mirrors and lenses

I imagined the laserstar was some massive Heat Radiator with a spacecraft strapped to it.

Thus the a flying lens/mirror would have higher acceleration and delta-vee. They could just let it melt after/as it did its duty.

Rick said...

I am not sure which is purple and which is green!

Here is my most recent take on kinetics v lasers. Short form, kinetics are most effective when deployed as a massive swarm of individually small target seekers. None too flexible, but could be cost effective as a defense.

A laserstar does indeed have honking big radiators, but so does any spacecraft with a high performance drive engine. But lens/mirror drones might still work out if they don't need a 'main drive,' but are simply deployed from the laserstar as it closes for action.

Anonymous said...

(SA Phil)

So the more-Dakka approach.

---
On the other hand-

How fast could you realistically get a missile in a Mid-future scenario?

What if you gave it a really long intial "push" with a high-power particle beam?

Essentially if you had a relatavistic missile/smart bullet With a final stage that allowed it to "steer" it would be in the Laser's threat window for a very short time.

leridan said...

"Our laser discussions (here and elsewhere) do tend to leave out the niggly little details of maintaining sub-microradian alignments in an environment that is a LOT less placid than the Hubble."

I've browsing the comments for just this. It may or may not be a problem when technology has also progressed enough to deliver the kind of lasers discussed here, but it does seem to be a problem: you'd better have pretty good stabilisation systems on your laser mount, otherwise the merest vibration in your ship will play hell with your fire control.

fire control systems are a critical aspect of modern warfare that is even more overlooked than logistics - this goes as far back as WWI where you can't really understand the maneuveurs of both sides during Jutland if you don't know about the differences between German and British FCS.

Rick said...

Welcome to another thread!

I tend to think that by the time we can build lasers in this class at all, we will be able to adequately solve the fire control / precision aiming problem.

But this is by no means a given, and anyone who wants to avoid uber-lasers in a midfuture setting can reasonably do so by invoking devils in the details.

Anonymous said...

"Purple vs Green:

So the problem from how I can see it is given lasers that are effctive out to 1/2 a light second or more, the fact they have "limited range" compared to kinetics is a non-starter given the actual likely relative closing speeds of any weapon that has a longer range than they do (such as a missile or other kinetic weapon)

Making the Laser the winner in the purple vs green (which one is which?)debate.

Unless you can make a suitably faster kinetic."

But this goes back to the eyeball frying contest: If I can get my laser unshuttered and targeted on your laser, the second you're shutter twitches I'll fry it (think the hollywood gunfighters each waiting for the other to move, but one already has his gun out and aimed). Then I can drop a torp out: either you get slagged by the massive kinetic impact, or you unshutter to kill its guidance. At which point I kill your laser and can destroy you at leisure. And while your laser can fire at half a light second, I can theoretically (with enough fuel/powerful enough engine) fire kinetics from across the star system: in enough quantities to overload your laser's ability to stop them.

That being said, to those who argue that a laser can't do anything about the mass of a kinetic missile, that's true, but that mass can't do any good if it doesn't hit. The laser can fry the guidance system/receiver (if the ship itself relays commands to the missile). Then it's a simple matter of nudging the thrusters and watching the missile harmlessly fly by.

If you forced me to take sides in PvG, I'd pick missiles. However, it's similar to discussion after WWI about the airplane: some said it would fight wars by itself, making armies and navies obsolete, and others said they couldn't make a significant difference, basically providing another form of reconnaissance and not much else. Both were wrong. Aircraft did make a significant impact on how wars were fought, but they did not replace conventional ground and naval forces. It will be the same way with lasers and missiles: both will have their proponents, but at the end of the day ships (or at least fleets) will carry both to take advantage of the strong points of both weapons.

On an unrelated note: earlier in the thread, someone mentioned that the size of the lens/mirror would limit the power of a laser routed through a turret, so a fixed lens would be more practical for anti-ship work. Would it be possible to mount an external mirror over a fixed lens, that could be adjusted to direct the laser beam to its target and get the equivalent field of fire as a turret?

-TDA

Anonymous said...

Another question: A ship armed with a centerline railgun is a gunship, a ship armed with lower-powered lasers for point defense is an interceptor, and a small (probably drone) ship with a short-range unguided kinetic shell for avoiding that point defense is a lancer. What is a ship that's armed with a high-power laser designed for long-range anti-ship firing called? A beamship?

Anonymous said...

IMHO, an intriguing trick to save mass may be to use the same components of the fusion rocket for the laser.
For example the relativistic electron beam of a Robert Bussard's QED rocket may be also used for a free electron laser. Instead of a massive solid undulator the ship can turn the plasma plume of the rocket in a very long plasma-ripple undulator preciding the FEL electron beam with short high frequency electron pulses.
This QED-FEL has no lens and mirrors, is focalized via magnetic nozzle field, uses the same propellant of the rocket ad it is regeneratively cooled.

I've made some research on Internet as an amateur SF writer, but I'm not an expert like you.
Do you tink in may work?

Thanks for attention
Luigi

Anonymous said...

Now, I'm no expert, but if we have mirror drones to redirect the mothership's laser, why don't we just use them to reflect the enemy ship's laser back at it? We even have reflectors on the moon specifically designed to reflect ranging lasers back to the source of the beam's emission.

To use this strategy effectively though, you'd have to know exactly when and where the enemy is going to fire far enough ahead of time to get the drones in position. This is all but impossible. However, you do know what the enemy is going to be shooting at: namely, your ship. So if you cover your ship's hull with reflectors, you won't need to know exactly what part of your ship they are going to shoot at; it doesn't matter, because the laser beam will be reflected back at the weapon it was fired from.

So, essentially, the enemy ship will take out their own lasers, and you are free to slice them to pieces with your own laser without fear of retribution.

Of course, the enemy will figure this out eventually, and cover their own ships in reflectors too. Which means that neither ship will be able to shoot the other without first destroying the enemy's reflectors. Otherwise, they would just end up destroying their own laser.

The easiest way to take out the reflectors would be with kinetic weapons. But, putting these on your ship would give the enemy something to shoot at. Even if you put it in a gun port, all the enemy ship has to do is target the port and wait for you to open it to fire the missiles, and then quickly fire their laser into the port. So we need some other method of delivering the kinetic weapons/missiles to the enemy ship.

This is where the lancer fighters would be handy: cover them with reflectors to protect from enemy fire, fly towards them, and launch the missiles at (relatively) close range to keep the enemy from having time to shoot them down.

As has already been pointed out, the best counter to the lancers would be more lancers with missiles. Since the lancers would be smaller, the missiles would need to be more accurate in order to hit them, meaning that they would need to adjust their course more, requiring more fuel. Missiles can also be dodged, especially by a maneuverable fighter. This means that in order for a missile to be able to hit a lancer before running out of fuel, it would have to be launched at closer range, potentially resulting in something similar to a dogfight as fighters attempt to avoid missiles and get a good firing solution on their opponents.

Byron said...

This is basically a version of mirror armor, which has significant practical problems. The first and simplest is that the mirroring only works against the first shot. Any mirror is not a perfect reflector, and will absorb some of the incident energy. At long ranges, this might be useful, but as the range closes, even that small amount of energy will be enough to melt the outer layer of the mirror, which in turn will destroy its reflective properties, leaving the vessel exposed to further shots. A normal mirror might have a reflectivity as high as 99.9%. A rough calculation suggests that for an aluminum outer skin, the beam intensity would have to be on the order of 40 MW/m2. Any other materials would require significantly higher intensities. This suggests that a pulsed beam would be more effective than a CW laser. The above calculation was based entirely on blackbody radiation, and ignores any number of complications. The author is unfamiliar with the response of reflective materials to laser radiation, but it does not seem outside the realm of possibility that the reflectivity could be significantly impaired by much lower rises in temperature. It is also unlikely that 99.9% reflectivity could be maintained on an operational spacecraft. This number reflects the maximum for conventional mirrors under laboratory conditions. The outer hull of a warship is far from the lab, and has to deal with things like solar wind and micrometeorites, which would likely limit the practical reflectivity to 99% at most. While it might be possible to put some form of protective covering on the armor and jettison it before battle, that limits the tactic to once per mission, and would require considerable effort to implement.
One might point out that the laser has to be focused by a mirror, and that the same mirror should be capable of being used as armor. The problem with this suggestion is that the mirrors used for lasers are not conventional mirrors, but dielectric mirrors. A dielectric mirror is made of numerous thin sheets of dielectric material, and is optimized for a particular wavelength and direction. Over that narrow band, reflectivity could be as high as 99.999%, but the mirror is significantly less reflective against any other incoming light sources. To use this type of mirror as armor, one would have to know the exact wavelength of an opponent’s weapons, and be able to control the direction on an engagement. Both of these are unlikely in practice, as any power will undoubtedly use slightly different wavelengths on different craft to defeat these tactics, and it is unlikely that one could control the engagement well enough to keep the enemy in the proper zone, certainly not likely enough to be worth the expense of fitting dielectric armor to a spacecraft.
(Excerpt from my space warfare paper).

Byron said...

Now, onto the problems with the retroreflectors themselves. Even if the problems described above could be overcome, there are significant issues with bouncing the beam back at its source. Quite simply, you can't put enough power back there to do anything of note. Taking the best-case scenario, that of a flat mirror perpendicular to the beam, the intensity when it returns to the firing ship will be only one-fourth the intensity at the target. If the target can maintain an optical surface under the impact of the beam, the target will have no difficulty doing so. Also, this is a set of ridiculously optimistic assumptions. The reflectors used on Apollo were corner reflectors and are used on many reflective objects. They do an excellent job reflecting in the general direction of the beam, but it's nowhere near precise enough for what you describe. It might somewhat dazzle them at long range, but that's about it. Corner reflectors have the advantage of reflecting no matter what direction the signal comes from. Other possibilities (such as the aforementioned flat mirror) do not, which means they need pointing capabilities on par with the laser itself, and have to deal with higher energy fluxes than the laser optics. Combine this with the issues described above, and I find it vanishingly unlikely that we'll see such things.

Rick said...

Apologies to commenter Luigi for totally missing your comment. Genuine comments to older posts were being lost in a sea of comment spam, which is why I had to go to Captcha.

I don't really know enough about FELs to judge the dual use idea, but I suspect there would be engineering problems, probably serious ones.

On reflectors I tend to agree with Byron.

Anonymous said...

The use of mechanical arrays, even active/adaptive optical ones is so jejune. Try a plasma window and use scalar RF. More powerloading at a less environment senstive bandpoint.
It gets you the beam agility of a phased array with the multilevel phase and charge loading of a 'disruptor' (tune to the nanno-resonant properties of individual atoms) weapon that has range equivalent to the radar you use to point it.
None of you mention the obvious problems of how to make your ships. I would suggest plasma sublimation of large asteroids with the fuse formed hulls being twice as dense when recooled and shaped via charged magnetic fields, also a scalar device possibility. Regardless, there will be some pretty hard 'up thru the gravity well' lofted payload limits on practical naval architecture for a long time.
Modern pulsed-impulsive laser weapons use both color (as frequency) stacking to get beyond the energy loss problem of burning through your own refractive plasma knockoff and something called 'X-wave' compression which is to say tucking in the lateral edges of the PEP so that it doesn't go lossy to self attenuation.
Both techniques -greatly- magnify the target-face retained power of pulsed weaponry.
Finally, you might want to consider the viability of boosted killer eggs that DON'T impact. But instead function somewhere between the level of an Excalibur (SDI system based atomic warhead satellite whose beryllium rods focussed X-Rays into multibeam DEWS).
And CAPTOR. A captive torpedo system which the USN used to control approaches to ports and the like. Could be laid like a mine but with vastly larger area coverage.
Since I don't see 30km/sec speeds as being likely in the near future of propulsion design (especially in NEO conditions) and I -do- believe in the black-constellation concept of stealthy small objects in space, it seems obvious to me that you put your throwaway guns ahead of your sensors as a screen and let them fire on the threats as they fly past in a moving beartrap fashion as the enemy flies up the vector in response to your approach. Indeed, did I want to take a world rather than just glass it, I would never come closer than a moon's shadow while I deployed sacrificial, non-capital, assets to reduce the defenses.

Anonymous said...

along comes the cruiser armed with a casaba howitzer and blows up a laser wielding capital ship the length of a football field with a single, well placed shot. or the missile frigate and it's long range fusion warhead equipped missiles. or the railgun toting destroyer, a single shot from which does about as much destructive force as a tomahawk cruise missile.

Unknown said...

Question, what about slanted armor? Atomic rockets says slanted armor can diffuse any laser no matter how powerful. I find this hard to believe. At what ranges might a MW-TW laser effect slanted armor?

What about x-ray lasers, or mythical gamma lasers? Would a high powered x-ray laser be effected by slanted armor? I don't see gamma rays giving a crap nugget about slanted armor as they fry all biologics and electronics on your ship.

Phillip Huggan said...

I've oscillated for and against various NASA faculties and am now pro-space over the long-term. Now, NASA is hackable by a dumb 2030 AI. But compared to Earth, at the Oort cloud and nearby stars it will be easier to stop robots from winning by blotting out the Sun, etc. You can watch payloads but technology is similar to a pandemic or hurricane and an unauthorized launch past GEO is a reasonable expectation. One solution is manufacturing (every few AUs or 50AUs) Grasers or X-ray lasers far enough away that diffusion limits their utility in attacking Earth. Ozone and/or O2 in ice near Earth could protect against Graser beam aimed at Earth and the ice could be melted by Lunar far-side lasers easily.
Earth can be patrolled by quantum-entangled microwaves against weapon R+D but for example, the smartest 3 Canadian Provinces have announced Crown funding of robotic or AI R+D without considering broader WMD consequences.

E S said...

@James Jordan: that Atomic Rockets post violates physics. For one, it erroneously assumes that the reflectivity of a material is constant no matter how powerful the incident laser. This is a false assumption, that has been known to be false for decades. See this pdf from 1980: http://www.dtic.mil/dtic/tr/fulltext/u2/a102005.pdf

Atomic Rockets has a lot of quotes these days that are based on a freaking video game called Children of a Dead Earth. As such, Atomic Rockets is currently many things, but a reliable source of information is not one of them.

Luck City said...

I am very enjoyed for this blog. Its an informative topic. It help me very much to solve some problems. Its opportunity are so fantastic and working style so speedy. laser tag gear

samia87 said...

Lidar scanner, Denver, Colorado is a good technology to do all type of video and camera leaning. This type of work is done by our organization. We are a dedicated worker. Visit us.

Ferlin Waffen said...

Hey what a brilliant post I have come across and believe me I have been searching out for this similar kind of post for past a week and hardly came across this. Thank you very much and will look for more postings from you Best Waffen kaufen deutschland service provider.

Saint Michael said...

Whoops, looks like some spam slipped through your filters, Rick!

Not a robot, just to clarify. I used to wish I was one... when I was like, seven. :)

On topic: Using a laser cannon to make the other guy expend their propellant is a good idea... outside of a planet's magnetosphere. In a sufficiently strong field (IE, Earth's) one could deploy an electrified tether to change orbits. Whether that's an effective "dodge" depends on techlevel and laser effectiveness.

The thermal issues will probably dominate laser usage and efficiency. As stated, any optics, mirrors, and heat sinks will probably not perform as ideally in the field as the lab models assume. That's why engineers laugh/cry at the promises made by the marketing department.

One thermal issue that might be dealt with; Some drives work on Open Cycle cooling, where the heat is dumped as the ship thrusts. Such a drive might be ideal for a laser star or other vessel that runs hot. So you'd only need one set of radiators.

technology said...

Scanners help telecommunications companies in at least three ways. 3d laser scanning survey Sydney